B corrections

¡Supera tus tareas y exámenes ahora con Quizwiz!

Ethidium bromide (EtBr) is a standard laboratory reagent used to indiscriminately label DNA for use in an electrophoresis gel. The EtBr molecule is both charged and hydrophobic, which allows it to intercalate between stacked base pairs. When intercalated in this manner, the conjugation state of the molecule is altered such that it vividly fluoresces. Suppose that EtBr is introduced into a PCR solution and allowed to remain throughout multiple thermal cycles. Fluorescent intensity would most likely:

answer-fluctuate multiple times throughout the procedure The question stem informs us that EtBr is fluorescent when positioned between stack base pairs. In other words it will fluorescence only when it may coordinate with an intact double helix. Since these helices are formed and denatured many times by the repeated heating and cooling of the PCR procedure, fluorescent activity is likely to fluctuate accordingly

Much effort has been exerted to optimize the timing and temperature ranges of the PCR thermal cycle, thus maximizing desired DNA yield and minimizing error. Poor yield in a PCR reaction would most reasonably be attributed to which of the following phenomena?

answer-magnesium stabilizes DNA double helixes Although magnesium is essential for PCR to proceed, it can occasionally coordinate strongly with the double helix and prevent complete DNA denaturation. This reduces the amount of substrate available for DNA polymerase and ultimately reduces yield

A researcher tries to express the human gene for insulin, screened from the human genomic library, in a bacterial cell. The translated protein fails to fold into its functional native conformation. What would best explain this misfolding?

answer-the genes in the human genomic library contain introns, which cannot be excised by the bacterial cell Bacterial cells cannot carry out post transcriptional modification of mRNA. Without this modification, introns will be translated along with exons, leading to incorrect folding of the expressed protein

In order for an enzyme obeying Michaelis Menten model to reach 1/2 of its maximum velocity :

answer: [s] must be equal to 1 Km [S] is the concentration of the substrate and Km implies that half of the active sites on the enzyme are filled. Different enzymes have different Km values. When all of the active sites are occupied, a reaction has reached maximum velocity. Therefore, Km is equal to the concentration of substrate when the rate is 1/2 of the maximum velocity

According to the classic Watson and Crick model of base pairing, the two complementary nucleobases comprising a pair are aligned such that they lie in the same plane, perpendicular to the long axis of the helix. However, recent applications of NMR spectroscopy have revealed substantial deviations from this ideal model. In some cases, the 2 nucleobases may transiently lie in 2 planes that are almost orthogonal. The angle between the 2 complementary bases is termed the propellar angle. A base pair with a large propellar angle will:

answer: reduce the overall melting temperature of the DNA duplex due to reduced hydrophobic contact The hydrophobic effect is the principle contributor to DNA stability. An increased propellar angle will reduce the hydrophobic contact between the tilted base pairs and adjacent bases in the stack. Ultimately this would weaken the attractions holding the DNA together, meaning that less heat will need to enter the system to melt the helix hydrogen bonding constitutes a secondary contribution to the overall stability of the helix

Calcitonin and PTH are ____ hormones

antagonistic

The blastopore, depending on the organism, may form ___ or ___

anus or mouth

An _____ is an inactive enzyme without its cofactor

apoenzyme

____ is sometimes known as "programmed cell death" which represents a naturally occurring process. In contrast ____ is triggered by an environmental stressor, such as extreme cold, injury, or lack of blood flow to an area. This promotes the death of otherwise healthy cells

apoptosis; necrosis

The interior of a protein is generally held far from the ____ environment outside of it

aqueous

A system that is far from equilibrium contains large energy gradients which ____

are able to perform work; the formation of large energy gradients requires localization or concentration of energy which results in decreased entropy

snRNA

are found in the nucleus and aid in splicing pre-mRNA

A low recombination frequency suggests that the two genes being measured _____

are physically close

Both collagen and elastin are found in the walls of larger vessels, especially ____

arteries

Smooth muscle is found in ____ and (to a lesser extent)_____, as well as _____ and large ____

arteries, veins, arterioles, large venules

One artery branches off into many ___, which then each branch to form countless capillaries.

arterioles

Example of codominance

as seen in human A and B blood types, two separate alleles are expressed simultaneously and contribute to the phenotype

A longer ____ period increases the likelihood of spread

asymptomatic

Class 1 transposon are known as _____ transposons as they involve the creation of a new copy of the transposable element. These elements first undergo transcription into RNA using RNA polymerase. As their name implies, they are the reverse transcribed back into DNA and placed in a distinct location elsewhere in the genome

"copy and paste"

two proteins produced from the same gene but have different functions is probably caused by transcription beginning at different _______ within the exons which encodes them

(alternate) promoter regions

Osmotic pressure involves the _____ to regions that are relatively solute rich

attraction of water

The most effective technique for sterilizing used laboratory materials is using an ____. An autoclave brings the materials to a temperature over 120 degrees celsius and a pressure over 2 atm, which is enough to kill anything

autoclave

____ agents bind to autocrine receptors on the same cell from which they are released

autocrine

All of the following are actions performed by macrophages EXCEPT: a)antigen presentations b)opsonization c)phagocytosis d)cytokine release

b) opsonization; opsonization is the process by which antibodies bind to and recognize antigens on the surface of a pathogen; the antigens then attract macrophages to phagocytose the invader a,c,d- all functions of macrophages

change of G degrees =

-RTln(Keq)

Km is the substrate concentration needed to reach ___

1/2 Vmax

The glycolytic pathway has ___ major points of regulation. In other words, only 3 of its steps are strictly regulated

3

DNA polymerase is only able to read DNA from the _____ direction and synthesize it from ___

3'-5'; 5'-3'

The ____ is the region of mRNA that is directly upstream from the initiation codon. This region is important for the regulation of translation of a transcript. As the "untranslated region" we can conclude that this region is not translated or only partially translated into protein

5'-untranslated region (5'-UTR)

Prokaryotic ribosomes have an overall size of ____, with individual subunits of ___ and ___

70s; 50s and 30s

Only prokaryotes have ribosomal subunits that are 30s and 50s with an overall structure of ___. While eukaryotes have ribosomes that are ___ overall, with individual subunits 40s and 60s

70s; 80s

758 of 1000 individuals have a dominant trait in which they cannot differentiate between the colors periwinkle and lavender. Which fraction of the population consists of people homozygous for this allele?

758 individuals described are those who express a dominant allele, meaning that they can either be homozygous or heterozygous. If we round 758 to 750, we can estimate 75% of the population has the inability to differentiate these colors; in other words P^2+2pq= 0.75 Using the equation p^2+2pq+q^2=1, we find that q^2=0.25 and q=0.5 Thus p=0.5, finally we can use this value to find p^2( the proportion of homozygous dominant individuals)=0.25

Eukaryotic protein synthesis occurs in the ___ ribosome. These are distinct in their subunit composition from ___ ribosome, which are found exclusively in prokaryotes

80s;70s

Conducting a dihybrid cross with two heterozygotes yields a Mendelian ratio of

9:3:3:1

Dyneins, motor proteins that are associated with microtubules, play a vital role in the transport of cellular components. How can the intracellular movements of dyneins be described? l. dyneins travel toward the center of the cell ll. dyneins move toward the minus ends of their associated microtubules lll. dyneins are involved in retrograde movement

A dyneins "walks" down its microtubule towards the minus end. The terminal is typically oriented towards the central region of the cell Additionally the act of traveling inward from the cell membrane is known as retrograde transport All 3 statements are correct

When ____ is present, the collecting duct is made water permeable through the introduction of aquaporin channels

ADH

low energy molecules include: ___, ___, ___ and ____

AMP, ADP, FAD, and NAD+

During a single round of glycolysis, one glucose molecule, 2 NAD+, two protons, and 2 ___ molecules are consumed. This process yields 2 pyruvates which are then converted to acetyl coA for insertion into the Kreb's Cycle. Glycolysis also creates 4 ATP and 2 NADH molecules

ATP

The TCA cycle is an alternative term for the Krebs Cycle. Like most biological pathways, the process is inhibited by its own products in a classic negative feedback. Products of the Krebs cycle include ___,____, and ___

ATP, citrate and NADH

One method by which cells drive nonspontaneous reactions is to couple them to other processes that involve high energy compounds. All of the following are species that contain high energy bonds except: a)ATP b)NAD+ c)creatine phosphate d)acetyl coA

ATP, creatine phosphate, and acetyl coA are similar in that they contain high energy phosphate or thioester bonds. In contrast, NAD+ is the oxidized form of NADH, an electron carrier. While it could be said that NADH possess a high energy bond, NAD+ itself represents a corresponding low energy state Alone, it is unable to provide the extremely negative change of G required to couple with a nonsponatneous process

___ is an allosteric inhibitor of phosphofructokinase while ___ reverses this inhibition

ATP; AMP

According to Le Chatelier's Principle, which glycolytic disturbances would force the metabolic system to reestablish equilibrium? l. increase concentration of glucose ll. decrease concentration of pyruvate lll. increase production of hexokinase iV. decrease availability of ATP

According to Le Chatelier's Principle, a stress exerted on a system typically promotes a shift to reestablish equilibrium according to Keq Such stresses include concentrations, volume, or pressure, temperature. Specifically, variations in the available concentration of product (pyruvate) or reactants (glucose and ATP) would disturb equilibrium Hexokinase is an enzyme that catalyzes the first step of glycolysis. Enzymes are neither reactants nor products and affect kinetics not thermodynamics

The absorbance of a protein solution is 3.37. If the concentration of the solution is known to be 4 M and the path length is 1 cm, which value best approximates the extinction coefficient?

According to the Beer-Lambert law, absorbance=eCL e represents the extinction coefficient C denotes the concentration of the solution L stands for the path length in centimeters solving for e gives a value of about .8

If a molecule of palmitic acid (C16H32O2) fully undergoes beta-oxidation, how many molecules of acetyl coA will be generate?

Acetyl CoA is produced in the final step of beta oxidation, when it forms alongside a new fatty acid that is reduced in length by 2 carbons. This process can than repeat. Thus to find the number acetyl coA molecules that will be produced from complete reaction of a fatty acid, simply divide the number of carbons by 2 Answer: 8

Throughout both the Krebs cycle and oxidative phosphorylation, how many molecules of ATP are formed per molecule of acetyl coA?

Acetyl coA is fed into the Krebs cycle to produce 3 NADH, 1 FADH and 1 GTP, the last of which is immediately converted to ATP. Each NADH is used to form 3 ATP while a single FADH2 is utilized to synthesize 2. In total 12 molecules of ATP are created

At the (-) end of a microtubule, depolymerization is prevented by:

All microtubules originate from microtubule-organizing centers, or MTOCs. These structures anchor the minus end of the microtubule to prevent its depolymerization. They also form the origin of the spindle apparatus during cell division

Preterm neonates commonly suffer from respiratory failure as a result of inadequate secretion of surfactant. When surfactant production is insufficient, alveoli tend to collapse in a condition known as atelectasis. In preterm neonate suffering from respiratory distress, which of the following behaviors would be most helpful? a)use of accessory muscles, including the trapezius and sternocleidomastoid b)grunting, which permits expiration against a positive pressure c)coughing, which serves to clear the airway of excessive secretions or obstructions d)maintaining an upright posture, which reduces the effects of pulmonary edema

All of these options have the potential to improve respiration, depending on the cause of respiratory failure. However only grunting will provide relief from atelectasis. This question stem tells us that this condition stems from the collapse of the alveoli. Expiration against a positive end-expiratory pressure (in other words, causing the alveolar pressure to be comparatively low) forces air into the alveoli, keeping them open and allowing gas exchange to continue a) this breathing technique effectively addresses conditions that hinder a rapid and efficient increase in gross lung volume, such as COPD. However, simply increasing the lung volume is unlikely to provide relief from an ailment that is alveolar in origin c) this will simply help if airway is osbstructed

Which of the following factors would be most likely to cause acetic acid to completely dissociate in aqueous solution?

Answer- continuous removal of protons from the solution in acid dissociation reactions, aqueous H+ is a product. Le Chatelier's Principle states that removing one of the products will cause equilibrium to shift toward the product side of the reaction -increasing pKa of a species implies that it becomes a weaker acid. Weak acids do not completely dissociate in solution

When oocyte is released from the follicle, it is surrounded by a layer of maternal follicular cells known as the cumulus oophorus. Which of these choices below is a plausible function of cumulus oophorus?

Answer- it provides vital amino acids and other molecules to support the fragile oocyte Since metabolic processes generate free radicals and other toxic byproducts, the oocyte must remain as metabolically inactive as possible to protect its haploid genome.

Joe conducts an experiment in his biochemistry class. First, he denatures Enzyme A under stringent conditions; afterwards, he removes these conditions and observes that the enzymatic activity of the protein is regained. Jon concludes the protein conformation: a)determined by the primary structure b)determined by the tertiary structure c)dependent on the chemical enviornment d)more than one of the above

Answer-D Even though the protein is first denatured, causing it to lose its higher level structure, its enzymatic activity is able to be restored once harsh conditions are removed. This indicates that a protein's conformation may be recovered even when only the primary structure is still present c- is also true because the protein was denatured in a certain environment (likely involving exposure to urea) and renatured in another

Vaccines routinely fail to provide lasting immunity because the antigen is rapidly cleared by local macrophages and neutrophils long before antigen-presenting cells can display it to the adaptive immune system. The effectiveness of a novel vaccine can be enhanced by:

Answer: co-administration of alum, a substance that triggers production of inflammatory cytokines The inflammatory process involves the recruitment of immune cells, including antigen presenting cells, to the affected area. This maximizes the chance of the antigen being picked up and taken to a nearby lymph node

Following puberty, the testes begin producing large amounts of testosterone. After production, the testosterone:

Answer: diffuses into the circulatory system and is transported around the body while bound to plasma protein Remember testosterone is a steroid hormone. As such, it can freely diffuse through cell membranes. Also as a hormone, testosterone is transported throughout the body in the circulatory system, not confined in the nuclei of the testes. Testosterone must bind to a transport protein since it is not hydrophilic and thus cannot simply dissolve in the blood plasma directly

Arrange the following in increasing order of compactness l. nucleosome ll. heterochromatin lll. euchromatin lV. DNA helix

Answer: lV, lll, ll, l. DNA helix is the least compact. Nucleosomes are the fundamental repeating units of eukaryotic chromatin, meaning they are the most compact. Chromatin is the primarily the combination of DNA and histones, the proteins responsible for packaging DNA into smaller volumes so they can fit inside a cell. There are 2 varieties of chromatin

During strenuous exercise, lactic acid builds in cells and causes the creation of a hydronium complex known as the Eigen cation (H9O4 +). If water molecules then experience hydrogen bonds attractions to the Eigen cations, this attractive force:

Answer: results in a semi-stable shell of water molecules around the hydronium A process known as "hydration" or "solvation" occurs when the attractive force of an ion molecule causes a thin shell of water molecules to surround it In the case of hydronium, each of the H atom attracts the O atom in an H2O molecule due to hydrogen bonding. These H20 molecules cause a shell of water molecules to surround the hydronium atom

In a certain population of wild coyotes, brown fur (B) is dominant over spotted fur (b) and approximately 80% of a given breeding population has brown fur. Over the course of two decades, a genetic change occurs such that nearly the entire population expresses the spotted fur phenotype. In this population:

Answer: spotted fur is the wild type The term wild-type refers to the traits an animal typically possess when found in nature. This usually refers to a dominant trait, but not always.

A researcher compares 2 antibodies that recognize the same antigen, even though they are made by different animal species. How will these antibodies differ?

Antibodies of different isotypes (IgA, IgG, etc) differ in their constant regions, as do those made by separate species. The constant region can be recognized by other immune components to further stimulate the response to a pathogen

___ refer to a domain of single-celled organisms that lack membrane bound organelles

Archaea

To be classified as an aromatic amino acid, a residue must: l. contain at least one planar ring, like threonine ll. include at least one conjugated system, like tyrosine lll. contain at least one ring that is marked by 4n+ 4 n electrons, like phenylalanine

Aromatic compounds are those that contain planar, conjugated rings and follow Hukel's rule, which stipulate that the system must contain 4n +2 pi electrons. Tyrosine a classic example of an aromatic amino acid fits all classifications-ll Threonine does not contain any rings; while phenylalanine is aromatic, the choice incorrectly states huckel's rule

In its phosphorylated state, a receptor tyrosine kinase is stable for quite some time before it ultimately dephosphorylated and ceases to propagate signals. Signal propagation can be achieved because:

As long as the phosphorylated tyrosine residue exist to provide a viable binding site, many downstream signaling molecules can bind, unbind, and move to exert their effects, allowing the process to repeat

At a pH of 6.0, which of the following amino acids would be found as neutral molecules? l.arginine ll.aspartic acid lll.alanine lV.phenylalanine

At a pH of 6, both alanine and phenylalanine will have protonated amino groups, which carries a +1 charge. Both molecules will also have deprotonated carboxyl group, which will be negative; this results in a net molecular charge of 0 Arginine contains a basic side chain with a pka>12. At pH 6, both it amino group and side chain will be protonated (+2) while its carboxyl group is deprotonated (-1) yielding a net charge of +1 Aspartic acid has an acidic side chain with a pKa of approximately 4. Under these conditions, the amino group will be protonated (+1) while the carboxyl group and side chain are deprotnated (-2) yielding an overall charge of -1

Shortly after engulfing bacteria, macrophages undergo a "respiratory burst" characterized by substantial ATP production in matter of milliseconds. This metabolic activity is so intense that macrophage usually die after engulfing 40-50 bacteria. The purpose of respiratory burst is most likely:

Bacteria must be promptly exposed to acidic conditions following phagocytosis. Otherwise, they could simply replicate within the phagosome and cause lysis of the immune cell. Macrophages can acidify their phagosomes to a ph approaching 1.5, which is extremely energetically demanding. As protons are pumped into the vesicles, the increased concentration of positive charge makes subsequent loading more difficult Answer: to provide energy for the rapid acidification of phagosomes via proton pumps

Maxine is researching a way to reduce the activity of an enzyme implicated in a number of diseases. She is attempting to engineer an antagonistic molecule that will competitively inhibit this disease causing catalyst. To do so, Maxine should create a molecule that most closely resembles the:

Because enzymes increase the rate of reaction by reducing activation energy, they have the highest affinity for (and best stabilize) the transition states of their associated reactions. Therefore, a transition state analogue can be a potent inhibitor while competitive inhibitors can also resemble the substrate, an enzyme generally has a higher affinity for the transition state

____ is synthesized in the liver then travels to the gallbladder where it is stored until needed.

Bile

The ____ characterizes the inverse relationship between binding affinity and plasma carbon dioxide concentration. An increase in CO2 concentration generally correlates to an oxygen deficiency. In such conditions, hemoglobin must be able to release oxygen when it reaches the tissues. As a result when the concentration of CO2 increases, hemoglobin decreases its affinity for ____. In contrast, low CO2 concentrations facilitate an increase in binding affinity

Bohr effect; oxygen

The crocodile death roll is an impressive hunting method in which the reptile drags its prey underwater and rolls at a high speed. Crocodiles also posses hemoglobin that can bind bicarbonate at the a1b2 interface. How might this feature enable crocodiles to remain underwater for extended periods of time?

By holding its breath, a crocodile can drastically raise its concentration of plasma bicarbonate. This increases the concentration of HCO3- which also occurs in humans who do not exchange enough CO2; the difference, however is that crocodile hemoglobin can bind bicarbonate If this action stabilizes the T or tense state of hemoglobin, it can decrease the affinity of the molecules for oxygen .O2 my then be quickly released Answer: Binding HCO3- stabilizes the T state of hemoglobin, allowing for more oxygen to be released into tissue

Somatostatin, aka growth inhibiting hormone, is the hormone that inhibits the release of ____

CCK

High cortisol levels negatively feedsback and inhibits ____ and ____ release

CRH and ACTH

____ walls are composed of one cell thick layer of endothelium to facilitate gas exchange

Capillary

Carbonic anhydrase catalyzes the conversion of water and CO2 to carbonic acid. This process is a crucial part of the maintenance of a buffered pH. Which metal cofactor is necessary for carbonic anhydrase to function? a)platnium b)iron c)zinc d)magnesium

Carbonic anhydrase is a metalloenzyme that catalyzes the reversible reaction between CO2 and H2O. The active site of this molecule contains a zinc molecule Iron is a metal cofactor that as part of hemoglobins heme group. This element both allows for hemoglobin's attachment to oxygen and facilitates cooperative binding; however, it is not present in carbonic anhydrase

____ contain gap junctions to allow for continuous propagation of a contraction signal

Cardiac muscle

Aspartic acid, lysine, and glutamine undergo cation exchange chromatography. How should they be ordered from longest to shortest retention time?

Cation exchange chromatography features a column with a negatively-charged stationary phase. This phase will interact with the positive lysine molecules, while repelling aspartic acid due to its negative carboxylate side chain answer: lysine, glutamine, aspartic acid

Phosphofructokinase, a metabolic enzyme, mediates the 3rd step in glycolysis. Its activity is allosterically controlled via a feedback mechanism. Which of the following metabolites is most likely an allosteric inhibitor of phosphofructokinase? a)glucose b)citrate c)glucose-6-phosphate d)adenosine monophosphate

Citrate is formed in the citric acid cycle. Because it is formed downstream of this step, it is likely the candidate to be involved in feedback inhibition

____ transposon produces an additional copy of transposon while ____ end results in the same total number of transposons

Class 1; Class ll or cut-and-paste

A congenital defect in the synthesis of the proteins cloudin and occludin is most likely to impact the function of:

Cloudin and occludin are the proteins that form tight junctions between epithelial cells. Therefore any organ that relies on tight junctions with be negatively affected if synthesis is impaired. If the intestinal lining is to properly absorb nutrients while leaving behind undesired materials, free diffusion cannot be permitted between the digestive ECM and the lumen of the digestive tract. Tight junctions seal gaps between epithelial cells in the intestine, allowing cells to selectively control what passes through the epithelium through the use of transmembrane transport proteins. Without tight junctions, nutrient concentrations would equalize due to diffusion through the epithelium, vastly decreasing the efficiency of the GI tract

Nicotinamide adenine dinucleotide (NAD) plays an important role in cellular respiration. In its reduced form, this species can transfer hydrogen to protein complexes that generate the H+ gradient required for ATP synthesis. NAD can be more specifically described as: a)coenzyme b)cofactor c)prosthetic group d)metalloprotein

Coenzymes are a subset of cofactors that tend to bind loosely to their associated enzymes. This type of molecule is known for transferring functional groups between species. NAD serves this function by donating its hydrogen to Complex 1, which is one of the 4 protein complexes responsible for generating the proton gradient during aerobic cellular respiration Cofactors is a broad category that encompasses both coenzymes and inorganic species.

Bats, birds and butterflies all developed wings in order to utilize the sky and all its associated advantages. What type of evolution is involved here?

Convergent evolution occurs when entirely separate lineages gradually appear more similar over time. Here bats, birds, butterflies are very distantly related, but all evolve to posses wings through different mechanisms. In the end, these species resemble each other despite their genetic differences

Lactose is composed of one ____ monomer and one ___ monomer

D-galactose and D-glucose

RNA polymerase is an enzyme that binds to the ___ and carries out transcription

DNA

Reverse transcriptase first synthesizes a single strand of ____ from the viral RNA template. It then synthesizes a second complementary strand of DNA using the first DNA strand as a template

DNA

Southern Blot assess for ___

DNA

transcription factors bind ____ and subsequently recruit RNA polymerase

DNA

______ creates RNA from a DNA template; this happens during regular transcription

DNA dependent RNA polymerase

The G2/M checkpoint ensures that ______ before the cell begins to divide

DNA has been replicated properly

The elongation process of DNA replication involves the addition of nucleotides to form a daughter strand complementary to the parent strand. ____ directly catalyzes this synthesis

DNA polymerase

___ extends DNA from 5' to 3', meaning that it adds nucleotides onto the free 3'end of the existing strand

DNA polymerase

Promotor region-

DNA sequence at a position where transcription of a gene will begin; this process requires a transcription factor or factors for initiation, followed by an RNA polymerase enzyme catalyzing transcription of DNA into RNA

Activators bind to ____ and help "recruit" promotors to start transcription. They are known to upregulate transcription in positive operons and in the negative lac operon, the cAMP activator protein (CAP) helps promote the transcription of the genes needed for lactose metabolism.

DNA sequences

Glycolysis, among other vital functions, serve to help regulate blood sugar. One hour after a meal, what is expected saturation status of hexokinase and glucokinase with glucose?

Directly after a meal, sugars are absorbed into the blood, resulting in high blood plasma glucose levels. Hexokinase has a much higher affinity for glucose than its isoenzyme, glucokinase. However, at high glucose concentrations, both enzymes should be saturated with glucose

A type of plant uses different insect pollinators depending on its height. Unfortunately, due to an increase in the use of pesticides, the medium-height pollinators has recently become instinct. What type of selection will occur in plant species over the next several generations?

Due to extinction of the associated insect species, medium height plants now cannot be pollinated. Over time, these plants will die out before they can reproduce, leaving only tall and short alleles to pass to the next generation. This represents disruptive selection, as only the extreme phenotypes are being evolutionarily favored.

____ are regulatory DNA sequences found in only eukaryotes. In prokaryotes, ____ serves as the regulatory DNA region instead

Enhancers; operator

Most, but not all, human cells can utilize aerobic respiration to create ATP in the presence of oxygen. Which of these cell types cannot perform this function and must rely on glycolysis? a)hepatic stellate cells b)epithelial cells c)erythrocytes d)mast cells

Erythrocytes or red blood cells do not contain a nucleus or mitochondria. Therefore, they cannot use Kreb's cycle or ETC to create ATP Liver cells would have no reason not to use aerobic methods of respiration b,d- both possess mitochondria and can utilize both Kreb's cycle and ETC

Some eukaryotic cells are covered with small ciliary projections used for adsorption, while others contain larger flagella used for propulsion. These cellular structures are composed of: a)microfilaments b)microtubules c)intermediate filaments d)myosin

Eukaryotic cilia and flagella are composed of bundles of microtubules. The answer is B -prokaryotic flagella are formed from the protein flagellin -microfilaments are composed of actin and found in the cytoplasm of eukaryotic cells as well as in muscle -intermediate filaments are less dynamic as actin or microtubular filaments; they are not involved in ciliary or flagellar movements -myosin is present in the muscle and aids in the process of contraction

____ and ___ are recycled between their oxidized and reduced forms. After NAD+ is reduced to NADH, this electron carrier may couple to another redox reaction, where it is oxidized and can act as a reducing agent for the other processes. FADH2 acts in the same way, cycling between its reduced state and FAD

FADH2 and NADH

____ stimulates spermatogenesis and thus leads to increase sperm production

FSH

The anterior pituitary produces ___, ___, ___, ___, ____, and ___

FSH, LH, ACTH, TSH, prolactin and growth hormone

A pharmaceutical compound targets and inhibits the carnitine shuttle used by cells. A human cell treated with this medication currently displays excess acetyl-coA inside the mitochondria and insufficient cytosolic acetyl-coA. What will be the most direct result of this circumstance? a) fatty acid oxidation is inhibited b)fatty acid synthesis is inhibited c)Kreb's cycle is inhibited d)stores of fatty tissue will increase in mass

Fatty acid synthesis takes place in the cytosol and requires acetyl coA, so this process will be inhibited if cytosolic concentrations of this molecule are too low a and c- since the interior of the mitochondria contains plenty of acetyl coA, fatty acid oxidation and Kreb's cycle should continue relatively unaffected d- fatty acid synthesis will stop but fatty acid oxidation will continue. In such a situation, adipose stores would decrease, if anything

____ does not yield ATP, it simply regenerates NAD+ required to continue glycolysis

Fermentation

In the U.S. population, the frequency of the allele for colorblindness is 8%. Which of the following is the frequency of colorblind woman and colorblind men in the population, respectively?

For a woman to be colorblind, she must get 2 copies of the gene. Since the frequency of the gene is .08, the odds of being homozygous for the gene is .0064 or .64% Answer choice: 0.64%, 8%

A researcher obtains pure glyceraldehyde 3 phosphate and uses it to mimic glycolysis and gluconeogenesis in vitro. If the researcher uses 6 moles of G3P, what is the maximum number of moles of pyruvate or moles of glucose that can be generated?

For every mole of glucose, 2 three carbon G3P intermediates may be produced, which then result in 2 pyruvate molecules as products. Thus following the ratio, 6 moles of G3P correlates to 6 moles of pyruvate Traveling in the reverse direction, gluconeogenesis would only yield 3 moles of glucose

Below is a table of pka values for lysine pka1--2.1 pka2--8.95 pka3--10.53 At which of the following pH values will lysine ionically bind to Na+ but not Cl- a)1.5 b)5.5 c)9.5 d)13.5

For the molecule to bind to Na+ but not Cl-, it must contain only negatively charged groups and no positive ones. This occurs when the entire molecule is deprotonated; the carboxylic group will be negative, while both amines will be neutral . For the amino acid to exist in this state the pH must be higher than the pka's of all groups d)

In the enzyme-catalyzed B12- dependent reaction that converts homocysteine to methionine, what most likely describes the role of vitamin B12?

From the question stem, we know that methionine synthase is involved in B12 dependent reaction that converts homocysteine to methionine. It can be inferred that B12 is a cofactor

Sucrose is a disaccharide that can be cleaved into D-glucose and D-fructose. Using ATP as a phosphate source, hexokinase can phosphorylate D -fructose, forming a product that may then enter glycolysis. Beginning with the entry of fructose-6 phosphate, what will be the net ATP production of one round of glycolysis?

Fructose 6 phosphate has already been phosphorylated once, it will enter glycolysis after step 2. This will save one ATP molecule that was previously required to phosphorylate glucose, increasing the net ATP production to 3 ATPs

___ is the state the cell will enter if it does not need to divide

G0

___ refers to the phase in the life cycle of the eukaryotic cell where it has exited the cell cycle. For example, once they are terminally differentiated, neurons remain in the G0 for the rest of their lives

G0

During the ____, the cell conducts protein and organelle synthesis at a high rate while the cell grows in size

G1

Which segment of interphase spans the longest period of time?

G1

The ____ checkpoint assesses whether the cell has enough organelles and is large enough to proceed towards DNA replication

G1/S

The ____checkpoint is crucial to ensuring that the recently replicated DNA was copied accurately; if not, mitosis will not occur

G2/M

____ is a chief inhibitory neurotransmitter

GABA

G proteins only activate when _____ is bound, a state that necessitates the release of GDP

GTP

____ acts to raise blood glucose levels; Glucagon stimulates fatty acid breakdown in adipocytes and this increases the total amount of fuel sources available in the blood

Glucagon

The net reaction of glycolysis is :

Glucose + 2NAD+ +2ADP---> 2 pyruvate+ 2 NADH +4ATP glucose, NAD+ and ADP are reactants, if any of these are deficient, glycolysis will be unable to proceed normally

Which of the following processes occur in the absence of oxygen? l. The Kreb's Cycle ll. Oxidative Phosphorylation lll. Glycolysis lV. Ethanol Fermentation

Glycolysis can proceed in the presence and absence of oxygen. Fermentation which forms either ethanol or lactic acid only occurs under anaerobic conditions, as it promotes the regeneration of NAD+ to allow glycolysis to continue Oxidative Phosphorylation requires proper functioning of the electron transport chain, for which oxygen serves as the final electron acceptor Kreb's does not directly utilize oxygen; however, it does not proceed when conditions are anaerobic

_____ are especially sensitive to antibiotics that target peptidoglycan

Gram positive cells

___ do not have an outer membrane while ___ posses an outer membrane

Gram positive; gram negative

____ bacteria posses a thin cell wall that does not fix to the crystal violet stain. Instead these cells are later stained pink with a counterstain

Gram-negative

____ organisms posses a thick peptidoglycan wall which attaches to crystal violet particles, preventing them from leaving the cell when it is subsequently washed. As a result these cells stain purple

Gram-positive

The _____ refers to the overlapping area between thin filaments in the center of the sarcomere

H zone

What are the histone proteins associated with the human nucleosome?

H1, H2A, H2B, H3, H4

Tyrosine Kinase Inhibitors are molecules that lodge in the active site of the tyrosine kinase enzyme. Glutamate is one of the essential residues on the kinase that hydrogen bonds with the substrate. In this reaction:

Hydrogen bonding occurs between a hydrogen atom (that is connected to N, O, or F) and an electrophilic center, such as N, O, or F. Under physiological conditions, glutamate's side chain is deprotonated and can therefore interact with an eligible hydrogen atom Answer: the -COO- from the side chain interacts with the -NH on TKI

The ____ describes the area between the ends of the myosin thick filaments and the Z line

I band

Tuberculosis begins with the colonization of the lungs by the bacterium M. tuberculosis. The lungs contain billions of alveolar macrophages that normally control the spread of airbone pathogens. M. tuberculosis lives within these macrophages, but is unable to enter alveolar epithelial cells. Consequently, a rapid and robust macrophage mediated response accelerates the spread of the infection. In vitro, the bacterium would be most infectious to: a)T-cells b)erythrocytes c)vascular endothelial cells d)neutrophils

If the bacterium can enter macrophages but not other somatic cells, it likely capitalizes on the natural entry mechanism offered by phagocytosis Although T cells and endothelial cells can phagocytize under certain circumstances (as can alveolar epithelial cells), neutrophils are professional phagocytes and would be rapidly infected by the bacterium

You desire to measure the amount of antigen in a blood sample using a western blot. Which of these reagents or molecules will be necessary I. sodium dodecyl sulfate ll. a reducing agent lll. a detection antibody IV. an agarose gel

In a western blot procedure, the proteins in the original mixture are first separated on an SDS-PAGE gel. In addition to SDS, a reducing agent is used to break disulfide bonds that may be present between cysteine residues. Next antibodies are used to detect particular species An agarose gel is generally more appropriate for DNA analysis than for protocols to deal with protein

It has been noted that certain cancers are linked with high rates of aerobic metabolism. A researcher desires to track the metabolic rate in a series of cell cultures. The researcher should monitor the formation of which product? a)H2 b)O2 c)ethanol d)CO2

In aerobic metabolism, CO2 is produced as a waste product H2 is unrelated to aerobic metabolism, O2 is a reactant in aerobic respiration and ethanol, like lactic acid is more closely associated with anaerobic pathways

Unlike humans, birds use ZW sex-determination system in which males have two Z chromosomes and females have one Z and one W chromosome. What is most likely the regulatory mechanism for potential imbalance in gene expression between male and female birds?

In humans, females have two of the same sex chromosome (XX) while males have one X and one Y. However, the X chromosome is much larger and carries significantly more genes than the Y version. To balance their genetic load with that of the males, one of each of female's X chromosome is inactivated. As far as we know, birds likely use the same mechanism but for the opposite gender. The male, who carries two Z chromosomes, must then inactivate one to balance his female ZW counterpart.

Which of these serves as an appropriate response to high blood sugar? l. increase glycogenesis ll.increase rate of gluconeogenesis lll. down regulation of glycolysis

In response to elevated plasma sugar levels, the body will aim to either use glucose or store it for later consumption. Thus, the rates for glycolysis or glycogenesis (creation of glycogen) will increase while the rate of gluconeogenesis will decrease

Tay Sachs disease is an autosomal recessive disorder that progressively destroys neurons in the brain and spinal cord. If q represents the Tay Sachs allele frequency in a hypothetical population, which of the following variables could be estimated using a population survey?

In the Hardy Weinberg equation, p and q represents the frequency of the dominant and recessive alleles found in a population. This equation is useful in determining the relative occurrence of genotypes in a given population, but to determine phenotypes, we must remember that each person obtains two copies of each allele As such the phenotypic outcomes in a population can be calculated by the following equation: p^2 +2pq+q^2 Answer: q^2

___ functions to decrease plasma glucose and promote its storage in the form of glycogen

Insulin

If the amino acids leucine and valine are run through the same ion-exchange chromatograph:

Ion-exchange chromatography separates compounds on the basis of charge. Depending on the stationary phase, either cations or anions will have longer retention times. in this situation, both compounds have non-polar side chains. thus using a charge based chromatography would be ineffective at separating them answer: it would not be possible to determine which peak corresponds to which amino acid

Higher affinity corresponds to lower ____

K

___ represents the catalytic rate constant, often called the turnover number. It is a measure of how many bound substrate molecules turn to product in 1 second

Kcat

Uncompetitive:

Km: decreases Vmax: decreases

Competitive:

Km: increases Vmax: stays the same

Noncompetitive

Km:same Vmax:drops

____ in response to stimulation by LH produce testosterone and other androgens

Leydig cells

Which amino acid(s) have side chains that typically include acidic protons at physiological pH? l. lysine ll. glutamic acid lll. aspartic acid

Lysine has a basic side chain while glutamic and aspartic acid have acidic side chains. At physiological pH, the carboxylic groups of glutamic and aspartic acid are deprotanated due to their low pKas. Only lysine will carry an acidic proton on its side chain

During most of the duration of an action potential, another stimulus, regardless of its strength, cannot cause the neuron to fire again. This is termed the absolute refractory period, and it can be attributed to:

Many voltage gated channels have two gates, an activation gate and an inactivation gate. When either is closed, transmission through the channel is blocked; however, inactivation gates are generally closed immediately after a channel has been open. This coincides with the duration of an absolute refractory period---from immediately after the sodium channels have been open through the repolarization phase Answer: the fact sodium channels are inactivated

Why might David suffer from a much more severe version of the disorder than either of the siblings?

Mitochondrial diseases are often associated with mitochondrial DNA, which is inherited through the maternal line. Thus, simply acquiring more mitochondria with the relevant mutation could result in a more severe form of the disease

Julia's family suffers from MELAS, a mitochondrial disorder that affects many of the body's systems. Her father David has the disease, as do her aunt Helen and uncle Vincent. David has a severe form of the disease, while Helen and and Vincent display fewer symptoms. Aunt Helen is married with only one child, Charles. Uncle Vincent has 3 children from different marriages: Michael, Cindy, and Sarah. From the given history, who is most at risk for inheriting a disease and should be tested?

Mitochondrial disorders are inherited maternally; in other words only affected mothers can pass along the disease. The only female who is said to have this condition is Helen. Helen is likely to pass the disease to her only child Charles.

____ neurons predominate in the CNS and the cerebrum specifically; their many dendrites allow them to integrate into a complex neural network capable of complicated processes

Multipolar

Complex 1 receives electrons from _____ and transfers electrons to Coenzyme Q

NADH

There are 5 electron carriers involved in the ETC: ____, ____,____,____ and ___ . Each carrier in the electron transport chain is only capable of carrying one or two electrons at a time

NADH-Q reductase, ubiquinone, cytochrome reductase, cytochrome c and cytochrome oxidase

During Krebs cycle, each molecule of acetyl coA forms 3 molecules of ___ and 1 molecule of ___

NADH; FADH2

____ is a reducing agent that is used in both fatty acid synthesis and glutathione reduction. In the latter pathway, NADPH relieves oxidative stress by aiding in the conversion of hydrogen peroxide to water

NADPH

The Na+/K+ pump helps the cell membrane maintain its membrane potential and creates a concentration gradient. It does this by moving ___ out of the cell and transporting ___ inward

Na+; K+

Which of the following enzymes must be present within the viral capsid of (-)RNA virus for successful infection and replication to take place?

Negative-sense RNA cannot be directly translated by host ribosomes to produce functional viral proteins. The (-) RNA must be used as a template for the production of (+) RNA by a viral RNA polymerase carried within the capsid. An enzyme that produces RNA from an RNA template is termed an RNA dependent RNA polymerase

A scientists exposes yeast cell to a chemical that inhibits the synthesis of flippase proteins. He then uses 14 C to track the eventual locations of newly synthesized lipids via radiolabeling. After the administration of the afromentioned inhibitor, where would the new, radioactive lipids are found? l. In the nuclear membrane ll. In the cytosolic side of cell membrane lll. In the extracellular side of the cell membrane

Newly synthesized lipids are originally inserted into the cytosolic leaflet of the ER membrane. The membrane of the ER is continuous with the nuclear membrane; therefore according to the fluid mosiac model, lipids are able to freely move between structures Answer: 1 and 2 Normally, synthesized lipids are transported from the ER to the cell membrane via vesicular transport. Here, lipids in the cytosolic leaflet of the ER membrane are made to comprise the cytosolic layer of the membrane of a vesicle, and later join the cytosol facing side of the cell membrane when the vesicle merges with this structure

Enzyme H, a macromolecule that plays a crucial role in metabolism, is under study by a team of researchers. They observe that the apparent Km of the enzyme catalyzed reaction is .76mM, in the presence of a noncompetitive inhibitor, Compound P. In the absence of any inhibitor, the Km of the same system is closest to: a).25mM b).5mM c).75mM d).76 M

Noncompetitive inhibitors do not change the apparent Km of their associated enzymes, though they do decrease the maximum rate of a reaction. .75mM is the closest choice to kM in the question stem

_____ detects the the presence of mRNA

Northern blotting

The pentose phosphate pathway is most likely active in cells of the: a)brain b)small intestine c) liver d)pancreas

One of the primary functions of the PPP is to produce NADPH which is used in fatty acid synthesis. Fatty acid formation from carbohydrates occur predominately in the liver

In glycolysis, what was the net yield of ATP per molecule of pyruvate produced?

One round of glycolysis requires two ATP molecules and produces a total of 4 ATP. In other words, glycolysis forms a net quantity of 2 ATP per molecule of glucose However, glucose is split into two molecules of pyruvate. For this reason, only 1 ATP molecule is made per unit of pyruvate

When hemoglobin binds to a single oxygen molecule, its affinity for O2 increases. This is an example of

Oxygen binding causes hemoglobin to undergo a conformational change that increases its affinity for additional O2 molecules . As a result, the hemoglobin-oxygen binding curve is sigmoidal in shape. this phenomenon is known as positive cooperativity

____ is routinely and exclusively used when amplification of a single copy or a few copies of a segment of DNA around a certain sequence is required

PCR (polymerase chain reaction)

____ is used to amplify DNA sequences of interests

PCR or polymerase chain reaction

___factors are secreted into the surrounding extracellular environment. From here, they diffuse to act on nearby cells

Paracrine

____ hormones are larger and relatively polar. As such, they are unable to enter a cell through its plasma membrane. Peptides bind to extracellular receptors.

Peptide

given the following polypeptide chain, in which direction will it migrate during electrophoresis at a pH of 8? Ala-Gly-Glu-Arg-Lys-Phe

Polypeptide chains contain two termini (C and N) which are negatively and positively charged, respectively at a pH of 8. The overall charge of the molecule depends on the charge of the side chains Acidic R groups (Glu) are negatively charged while basic R groups (Lys and Arg) are positively charged Therefore the overall polypeptide charge is +1 and the molecule will travel to the negative pole Answer: it will migrate towards the negative pole

_____ tend to induce misfolding and aggregation of endogenous cellular, forming highly stable amyloid fibers.

Prions

____ carries deoxygenated blood to the lungs

Pulmonary artery

___ receive oxygenated blood from the lungs and carry it back to the heart, as such they likely contain the blood with the highest partial pressure of oxygen gas

Pulmonary veins

____ is associated with ribosomes; makes secretory and membrane proteins ____ makes lipids

RER SER

_____ forms RNA from an RNA template. Such an enzyme does not exist in eukaryotes though it is found in viruses

RNA dependent RNA polymerase

Ribozymes are ___.

RNA molecules that are capable of catalyzing chemical reactions, including RNA and DNA ligation and peptide bond formation

Promotors are essential to eukaryotes and prokaryotes since they function as the DNA sequences to which ____ initially binds

RNA polymerase

The table below shows the pKa values for an unknown amino acid, "X" Group pka 1 2.10 2 4.07 3 9.47 For this information, it must be true that: a)group 2 is not the R group b)group 3 is the amino group c)the unknown amino acid could be Met d)at physiological ph, X has a positive charge

Regardless of the identity of the R group, the pka of the amino group is always 9, and the carboxyl group is always 2 So group 3 is obviously the amine group and group 1 is the carboxyl group. This means group 2 is the R group. the answer is b) c)-methionine has a neutral side chain. The unknown amino acid "X" appears to have an acidic R group d) at physiological pH, X is likely to have a -1 charge. Since the external pH is greater than group 2

__ is a measure of distance traveled over solvent front

Rf

Cell replicates its DNA during the ___ prior to meiosis l, but no additional replication precedes meiosis ll

S phase

DNA synthesis only occurs during ___ of the cell cycle

S phase

DNA replication occurs during ___ of the cell cycle; the nuclear envelope reforms during ___ of the cell cycle

S phase; telophase (during mitosis)

How can a scientist running an SDS-PAGE procedure assure that the difference in running distance on the gel is due to the length of the peptide and not its shape? a)polyacrilmide gels have larger pores than agarose gels, so this is not a concern b)the electric field used in SDS-PAGE protocols is sufficiently high, so this is not a conern c)detergent is added to the running buffer d)a reagent is added with the intent of denaturing the proteins primary structure

SDS is a detergent, meaning that it has both hydrophobic and a hydrophilic end, like soap. This substance coats the protein and destroys hydrophobic interactions, largely denaturing all but the primary structure. As a result, the protein can move through the gel in a linear fashion, removing its shape as a potential confounding variable a,b- the idea that shape might skew the results of the polyacrilmide gel procedure is certainly a major concern d- SDS does not denature primary structure

____ separates proteins based on their mass

SDS-PAGE

During spermatogenesis, the main function of ___ is to nourish the developing sperm cells. These cells are located in the epithelial lining of the seminiferous tubules and are activated by FSH

Sertoli cells

Sperm is produced by the ____ of the seminiferous tubules

Sertoli cells

A urological researcher is able to extract minute samples from particular regions of a mammalian nephron. One particular sample, Sample A, contains cuboidal epithelial cells that appear to be lined with microvilli. The most likely origin of Sample A is the: a)collecting duct b)glomerulus c)descending loop of henle d)proximal convoluted tubule

Simple cuboidal epithelia are well suited for solute transport and absorption. In addition, the presence of microvilli should remind you of the small intestine where nutrient absorption is the main function. Of the choices given the only region of the nephron heavily involved in absorption is the proximal convoluted tubule. Cells lining this structure must reabsorb glucose, sodium, amino acids and other vital solutes while also permitting the secretion of ions and toxins

p-nitrophenol has a pka of 7.16. Which of the following most accurately approximates the ratio of the conjugate base to p-nitrophenol in the pH 11 stop solution?

Since we are asked about the ratio of conjugate base to its corresponding acid in solution, this question relates to buffers. The Henderson-Hasselbach equation can be used: pH=pKa+log [conjugate base]/[acid] 11=7 +log [CB]/[A]--->4=log [CB]/[A] 10^4=[CB]/[A] Final answer: 10,000:1

____ which are derived from cholesterol are small and non-polar. These features allow them to transverse the membrane without a channel; additionally they are long-acting and often alter gene expression in the nucleus

Steroid hormones

In an isolated cave, two bat species are discovered and found to share a distant ancestor. The ears of the two species seem to be adapted to find different sizes of prey using echolocation. When crossed, individuals of the distinct species are unable to produce a virile litter. This situation exemplifies:

Sympatric speciation is that which occurs without a physical barrier. A population diverges into two separate species in a single cave certainly falls under this form of speciation

At excess levels, which of the following could serve as an activator of the TCA enzyme citrate synthase? a)ADP b)citrate c)succinyl-coA d)NADH

TCA is another name for the Kreb's cycle or citric acid cycle. Increased concentration of ADP signals that more ATP is needed. Therefore, the citric acid cycle enzyme in question will be activated by the presence of ADP Citrate is the product of the enzyme citrate synthase. Through negative feedback, citrate accumulation would inhibit further citrate production Succinyl- coA is an intermediate of the citric acid cycle. Excess accumulation of cycle intermediates would serve as a signal that enough citric acid cylces have been run NADH is a product of the citric acid cycle

Asthma is a potentially life threatening condition characterized by the sustained contraction of bronchiolar smooth muscle, resulting in bronchoconstriction. Which of the following medications would most likely be effective in the treatment of asthma? a)terbutaline, a beta receptor agonist often administered to prevent preterm labor b)carvedilol, a beta receptor antagonist that prevents the binding of norepinephrine to cardiac muscle cells c)flexeril, a muscle relaxer that is effective in treating pain caused by skeletal muscle spasms d)pilocarpine, a powerful activator of parasympathetic activity

Terbutaline is effective in preventing uterine contractions that lead to pre-term labor. In other words, it is known to relax smooth muscle. Thus, it is reasonable to assume that this drug could act similarly to relax smooth muscle that line the bronchioles. Carvedilol antagonizes beta receptors which lead to further constriction in the lungs. For example, epinephrine (a beta receptor agonist) acts to dilate or widen the bronchioles. Thus an inhibitor of the same beta receptors could not possibly facilitate bronchodilation Although fexeril effectively causes the relaxation of skeletal muscle, we have no idea whether it would act similarly upon smooth muscle Remember, sympathetic not parasympathetic activity enables the relaxation of bronchiolar smooth muscle. Activation of a parasympathetic pathway will likely further cause constriction of the bronchioles

In the process used to create cDNA, the ssDNA produced by reverse transcriptase can be made into dsDNA through treatment with DNA polymerase 1. In this case, no primer is required because

The 3' end of the ssDNA often loops back onto itself due to complementary base pairing. This serves as a primer for DNA polymerase 1, providing the free 3'-OH group necessary for the enzyme to initiate synthesis

A clinical biologist is viewing a tissue sample from a patient with an osteosarcoma. The sample displays multinucleate cells and a "striped" appearance and appears to be innervated with somatic nerves. These cells most probably derived from which germ layers?

The description in the question stem most closely correlates to skeletal muscle. The tissue type along with the skeletal muscle system, the cardiovascular system, and the kidneys derive from the mesodermal layer

The amino acid with the chemical formula C5H11NO2S:

The given formula is that of methionine, a residue that contains thioester functionality as part of its side chain. Since it does not possess a free -SH group, it cannot establish disulfide linkages with the R groups of other amino acids Answer: it is not able to form disulfide bonds with other residues, because it does not possess a free thiol group

Which of the following segments of amino acids would be most likely to be found in the membrane spanning domain of the sodium channel in a nerve axon? a)DDR b)EVE c)LAD d)LIV

The inside of a membrane is hydrophobic so the membrane spanning domain of a protein will likely consist of of more hydrophobic amino acid residues Choice D represents 3 amino acids with hydrophobic side chains: leucine, isoleucine, and valine

Following fertilization of an ovum by a sperm cell, 4 cell divisions occur with virtually no cell growth. The resulting 16 cell division is known as the:

The morula is a spherical collection of 16 cells that is the same size as the original zygote. Further division results in the blastula, a hollow ball of cells. The blastula then develops into a gastrula, which is an invaginated structure containing 3 germ layers

In eukaryotic cells, where is the highest concentration of phosphorus?

The nucleus contains most of the DNA of a eukaryotic cell (some is also found in the mitochondria). The DNA backbone is made up of phosphate groups and deoxyribose sugars

Which of the following correctly identifies the order of development from fertilization to differentiation of the GI system?

The order of differentiation is zygote, morula, blastocyst, gastrula, 3 germ layers, endodermal layer, lining of archenteron (hollow gut cavity), GI system

Which cytoskeletal component forms the majority of the outer layer of skin?

The outer layer of skin is made up of keratin accumulated in dead cells. Keratin is an intermediate filament that has great strength

In which phase of the cell cycle would it be reasonable to expect the highest activity of the pentose phosphate pathway? a)G1 b)S c) G2 d) M

The pentose phosphate pathway produces ribose-5 phosphate, a compound that plays an important role in nucleic acid synthesis. During the S phase, DNA replication is ongoing and the demand for nucleic acids is expected to be high

Freckled skin is a recessive trait. If the incidence of freckled individuals in the population is about 36% , what percentage of people are heterozygote for the alleles for freckles?

The people who express the traits for freckles must have two copies of the allele. We can therefore solve for q, knowing that .36 represents q^2. Once we have found that q=.6, we can use p+q=1 to determine that p=.4. Finally the carrier rate must be expressed by 2pq=2(.4)(.6)=.48

The formation of which of the following products serve as the main goal of the pentose phosphate pathway? a)NADPH b)ribulose-5 phosphate c)glyceraldehyde 3 phosphate d)fructose 6 phosphate

The primary function of the pentose phosphate pathway is to generate large quantities of NADPH for use in anabolic reactions. Without NADPH, numerous critical pathways would be hindered, including nucleic acid and fatty acid synthesis. This would directly impact the viability of the cell b- the PPP produces ribose 5 phosphate not ribulose 5 phosphate c,d- These products link the pentose phosphate pathway with glycolysis, gluconeogenesis, and aerobic cellular respiration. However, their production, does not serve as a main function of the pathway

Apoptosis is a sequential process that can be divided into 3 distinct phases: initiation, commitment, and execution. The commitment stage is marked by a measurable dissipation in the mitochondrial proton gradient, owing to the permeabilization of the outer mitochondrial membrane. The execution stage features the activity of cysteine proteases that degrade vital cellular components. Cysteine protease activation is most directly triggered by: a) the release of cytochrome c into the cytosol b)the disintegration of the nuclear lamina c)the presence of DNA fragments in the cytosol d)the binding of TNF proteins to extracellular death receptors

The question mentions that the mitochondrial membrane is made more permeable during the commitment stage. This results in the release of cytochrome c into the cytosol. Cellular apoptotic proteins then bind cytochrome c then and effect capase activation b,c- we are told that capases degrade vital cellular components which could theoretically include the nuclear lamina. this would result in the release of DNA into the cytoplasm. However, this would only be possible if capases already had been activated, so this must occur further along in the pathway d-reception of the apoptotic signal would most likely occur early in the process. In other words it would probably precede the commitment phase

Some bacterial toxins are able to bind with high affinity to MHC, displacing whatever peptide was previously bound. As a result, the immune response is systematically activated without a clear target. The resulting pathology would most resemble:

The question stem describes an immune reaction in which the immune reaction responds to all cell types that express MHC (essentially all somatic cells). This closely resembles anaphylaxis, an extreme systemic allergic reaction that can be fatal

Protein sequencing and Western blotting performed on the mutant and wild-type AAT protein produce near identical results. If this is accurate, then which of the following mutations of the AAT gene is most likely responsible for the polymerization of alpha1-AT in the liver? a)missense b)nonsense c)frameshift d)silent

The question stem tells us that the sequence and size (via western blotting) of the 2 proteins are nearly identical. The passage states that the AAT mutation responsible for alpha1-AT polymerization involves a single purine-purine substitution A single nucleotide substitution that codes for an altered protein product without also altering the length of the protein is likely a missense mutation

Atorvastatin is a drug used to treat hypercholestoremia. Which of the following is most likely the mechanism of action by which atorvastatin reduces serum cholesterol?

The rate limiting step of a biosynthetic pathway is typically the best target for regulation, both exogenous and endogenous. Altering activity of rate limiting enzymes produces the fastest and most pronounced change in metabolic output Answer:Atorvastatin inhibits HMG-CoA reductase, the rate limiting enzyme in cholesterol biosynthesis pathway

Which are the following conditions is/are most likely to cause substantial decrease in tidal volume? l.acute asthma ll. late pregnancy lll. narcotic overdose lV. pulmonary embolism

The substantial volume occupied by the gravid uterus does in fact decrease the ability of the lungs to expand, reducing tidal volume. Overdose on a narcotic decreases respiratory drive, resulting in a decrease in rate and depth of breathing Asthma results from the narrowing of bronchioles, which inhibits the effective movement of air into and out of the lungs. This condition does not alter tidal volume Pulmonary embolism is a circulatory, not a respiratory, problem. It does not disrupt the ability of the lungs to expand or conduct air. Although they are in imminent danger, patients with pulmonary emboli display clear lung sounds with effective movement of air

While the heart pumps blood to both the systemic and pulmonary circuits, coronary arteries supply blood to the heart itself. A cardiothoracic surgeon isolates a sample of myocytes from the inner lining of one of the vessels. This sample most likely contains: a) cardiac muscle b)endothelium c)skeletal muscle d)smooth muscle

The term myocyte means muscle cell so we simply need to identify the type of muscle that lines the coronary vessels. Arteries (as well as arterioles, veins, and venules) contain a layer of smooth muscle in their walls. This allows for processes such as vasoconstriction and vasodilation. a) the tissue of the heart is composed of cardiac muscle, but the question stem references the lining of the artery b) this is tempting, as all blood vessels are coated in endothelial cells; however, the surgeon is said to have isolated myocytes and so we can only know for sure the sample contains muscle

which of the following best explains why bile is effective in digesting lipids? a)its acidic b)its amphipathic c)its amphoteric d)its hydrophobic

This question asks us to determine what best explains the effectiveness of bile in lipid digestion. We know that bile works to form micelles and is able to do this because it has both hydrophobic and hydrophilic portions. This is what "amphipathic" means

In miRNA-directed gene silencing, a small RNA binds to an mRNA and directs degradation of the mRNA or prevents translation of mRNA. Which of the following terms describes the process through which binding occurs?

This question describes hybridization; hybridization describes a process of binding through complementary nucleotides

Lack of bile acids can lead to deficiency in which of the following? a)cholesterol b)glucose c)vitamin k d)glycine

This question is asking us to determine which of the 4 options is fat soluble and thus will be taken up from the small intestines with the help of bile. We should know that vitamin K is a fat soluble vitamin (as are A, D, and E)and thus its absorption depends on fat's absorption from the small intestines a- a lack of bile salts would not necessarily mean that cholesterol is deficient. If cholesterol was not able to be converted to bile salts, the lack of bile acids could correlate to an overabundance of cholesterol b-glucose is not a fat soluble molecule d-glycine has other factors that allow for its absorption

Thyroxine is a hormone produced by the thyroid in a reaction that uses a tyrosine substrate. This hormone affects virtually every cell in the body by binding to its cytosolic receptor and translocating into the nucleus. Compared to a steroid hormone, thyroxine:

Thyroid does not produce any steroid hormone. However, both steroid hormones and thyroxine act by binding cytosolic receptors and entering the nucleus to alter gene transcription Answer: differs in its tissue of origin and structure, but shares a similar mechanism of action

Describe a reason why a electrical signal would need to travel more slowly through the AV node?

To allow blood from the atria to enter the ventricles before their contraction

DNA molecules are not able to function as enzymes. T or F?

True

Cell cycle checkpoint genes are a type of ____. They prevent premature cells from undergoing mitosis.

Tumor suppressor gene

In eukaryotes, the three stop codons are:

UGA, UAG and UAA

In an experiment in physiology lab, Jonathon poisons a laboratory animal by inhibiting a particular enzyme. He observes that the animal displays hypotension, a lowered heart rate, and prolonged skeletal muscle contractions. The enzyme targeted was most likely: a) a nicotinic receptor b) a calcium channel c) a muscle potentiator enzyme d) acetylcholinesterase

Upon downregulation of acetylcholinesterase, the concentration of acetylcholine will increase. Remember in the parasympathetic nervous system, acetylcholine lowers heart rate and blood pressure by acting upon muscarinic channels in the cardiac muscle. However, ACh also causes muscle contraction at the neuromuscular junction Targeting either nicotinic receptors or calcium channels would negatively impact skeletal muscle contraction c- this is nonexistent class of enzymes

What is the value for Km if [s]=25 M, Vmax=8m/s and Vo=4 m/s?

Use the Michaelis Menten Equation: Vo=Vmax[s]/Km+[s] Since the initial velocity is 1/2 its maximum, we can substitute: 1/2 vmax for Vo 1/2 Vmax=Vmax[s]/Km+[s] which simplifies to 1/2=[s]/Km+[s] solving for km shows it equals [s] answer:25 M

___ is not dependent on substrate concentration. It increases only if more enzyme is added

Vmax

Michaelis Menten Equation:

Vo=Vmax[s]/Km + [s] an increase in any value in the numerator yields an increase in initial velocity while an increase in denominator value such as Km decrease initial velocity

In a particular biological system, 10 mM of substrate is present. If the initial velocity of the associated enzyme catalyzed reaction is 1/8 its Vmax, km is equal to:

Vo=Vmax[s]/Km+[s] 1/8vmax=vmax[s]/Km+[s] 1/8=[s]/Km+[s] Km=7[s] or 70 mM

A researcher is attempting to create an artificial cell membrane that retains its fluidity at extremely low temperatures. Which feature should he incorporate in this membrane?

We are looking for characteristics of a membrane that increases fluidity, especially in cold environments. Unsaturated fatty acids increase fluidity in general due to the presence of "kinks" in their structure. Cholesterol and similar molecules act as a type of buffer system for membrane fluidity. Large amounts of cholesterol decrease fluidity at high temperatures but increase fluidity at low temperatures

Which of the following is the isoelectric point for alanine?

We know that alanine has no acidic or basic side groups, (but like all amino acids) it does have an amino terminus and a carboxy terminus that contribute to the pI calculation. A basic understanding of pI leads us to the recognition that in an amino acid like alanine, we can simply take the average of the pKa's of the amine and carboxy termini. This gives us (9+2)/2=5.5

The rate of actin polymerization in a solution was observed and plotted against the concentration of actin available. All values were normalized so that both the final concentration and the final rate were equal to 1 Actin concentration Polymerization Rate .2 -1 .4 -.5 .6 0 .8 .5 1 1 According to this data, what is the critical concentration of actin under these conditions?

With regard to actin, the critical concentration is the point at which no net polymerization or depolymerization occurs. This effectively represents the concentration where the length of the polymer is stable. The table shows that at the concentration of .6, the rate of polymerization is 0. This is the critical concentration

Kleinefelter syndrome

XXY

The ____ is the name given to the junction, or "border" at either end of a sarcomere. These structures are orthogonal to the direction of contraction and do not expand or become smaller during muscle movement

Z line

Polycistronic mRNA carries several open reading frames each of which is translated into a polypeptide. these polypeptides usually have related function (they often are the subunits composing a final complex protein), and their coding sequences are grouped and regulated together in a regulatory region, containing a promoter and a operator. Most of the mRNA found in ____ are polycistronic

bacteria

____ are a form of bacteria-infecting virus and are not considered living organisms at all

bacteriophage

____ is a virus that reproduces using the host prokaryotic machinery

bacteriophage

Which is least likely to be lysed by a T cell upon presentation of an antigen in the context of MHC? a)a dendritic cell representing a peptide of E.coli b)a pharyngeal epithelial cell presenting a peptide from M. tuberculosis c)a tumor cell presenting a peptide derived from a mutant fusion protein d) a grafted skin cell from an unrelated donor presenting an intracellular peptide

a) dendritic cells present antigen to helper T cells to activate the adaptive immune system; they are not lysed by the T cell to which they present b,c- both of these cells are presenting antigens derived from proteins unfamiliar to the host immune system. Healthy T cells will recognize these antigens and lyse the presenting cells d) although the cell may be presenting a normal peptide shared by host cells, MHC varies among individuals. T cells recognize the amino acid sequence of both the antigen and surrounding MHC domain. If either is unfamiliar to the T cell, the presenting cell would be lysed

The pancreas releases enzymes into the duodenum and releases ____ to raise the pH of the duodenum

bicarbonate

The 3' end of a tRNA molecule is responsible for

binding the amino acid

____ are common in special sensory organs and most commonly remembered as components of the retina

bipolar neurons

The ____ is a fluid filled central region present in the blastocyst during mammalian embryogenesis

blastocoel

the _____ consists of an inner cell mass, along with an outer cell layer called the trophoblast which surrounds both the inner cell mass and the blastocoel

blastocyst

Endothelium is a type of epithelium that lines the surface of _____ and ____, forming an interface between circulating blood or lymph in the lumen and the rest of the vessel wall

blood vessels and lymphatic vessels

A protein from the urine of a chronically ill patient was isolated and run on a SDS-PAGE gel. The result displayed on the gel was a long smear instead of a crisp band. These results can most likely be attributed to the fact that: a)the protein was partially degraded b)SDS denatured a portion of the primary structure c)the protein being analyzed was composed of multiple subunits of different molecular weights d)multiple proteins of distinct molecular weights were present in the sample

a: only partial degradation would account for the smear described in the question stem. This would break the protein into a very large number of lightly-colored bands, which appear to blend together on the polyacrilimide gel b- while denaturation of the primary structure might explain the results, SDS is unable to accomplish this c,d- these scenarios would result in multiple well defined bands, not a single smear

Ligaments connect ___ to ___ while tendons attach bone to muscle

bone to bone

As cells progress from totipotency to pluripotency to multipotency, their lineage potential decreases, meaning they lose their ability to give rise to a _____. Additionally their DNA becomes increasingly methylated. Remember DNA methylation decreases transcription rates

broad range of tissue types

The ____ lasts nearly the entire duration of an action potential during which time a second action potential can not be generated. During this time voltage gated sodium channels are "inactivated". If the "inactivation" gate is closed, the channels are inactivated

absolute refractory period

___ is formed from the breakdown of acetoacetic acid. Up to 30% of the product can be excreted from the body in the urine or through mechanisms such as exhalation. Unlike the other ketone bodies, acetone cannot be converted back to acetyl coA

acetone

The human gluconeogenetic pathway can utilize all of the following molecules except: a)oxaloacetate b)pyruvate c)acetyl coA d)alanine

acetyl coA cannot be directly incorporated into gluconeogenesis alanine can be converted to pyruvate in the liver then fed directly into the pathway

At physiological pH, ___ amino acids are negatively charged since their side chains contain a COO- group; ____ amino acids are positively charged at physiological pH

acidic; basic

For skeletal muscle to relax, the conformation of myosin must change as ATP binds the myosin head, decreasing its affinity for ___, and breaking the cross bridge between thick and thin filaments

actin

The protein component of microfilament is ___

actin

Macrophages must undergo rapid _____ during phagocytosis. If a macrophage cannot engulf bacteria in this manner, its overall function would be considered very impaired

actin reorganization

In a positive inducible operon, the ______ is incapable of binding the DNA during the basal state. Upon binding of the ___, the activator can undergo a conformational change that allows it to bind DNA and initiate transcription

activator protein; inducer

Inhibitors deactivate ____ in positive repressible operons and decrease the level of transcription

activator proteins

Zymogens require covalent modification via hydrolysis, significant structural rearrangement, or both before they can function as an _____

active enzyme

The attachment of hemoglobin to a single oxygen molecule promotes a change in the position of the central iron ion in the heme group's porphyrin ring. This facilitates the binding of _____; this is called cooperative binding

additional oxygen molecules

Primase

adds primers to the unwound DNA to give DNA polymerase a free -OH group to attach to

_____ separates proteins based on their interactions with specific ligands

affinity chromatography

The condensation of of 2-3 carbon molecules into a larger 6 carbon structure is mediated by an ____

aldolase enzyme

galactose is a 6 carbon ___

aldose

____ increases H20 and Na+ reabsorption from kidney while exchanging K+ ions for Na+ ions.

aldosterone

The adrenal gland can be subdivided into two parts, the adrenal cortex and adrenal medulla. The cortex produces ____ and ___ while the medulla secretes ___ and ____

aldosterone and cortisol; epinephrine and norepinephrine

The renin-angiotensin system results in the secretion of ____. The steroid hormone regulates blood pressure by increasing reabsorption of sodium, thus increasing blood pressure and decreasing urine output. ____ is another main effect of aldosterone, as it also increases blood pressure

aldosterone; vasoconstriction

Sigma factors are needed to ___

align RNA polymerase with the promoter sequence

Since CO2 in the plasma directly leads to the formation of carbonic acid, low CO2 concentration correlates to ____ or ___ blood

alkaline or basic

Genetic drift is simply the change in ____ due to random processes. Specifically random chance plays a role in determining which alleles are inherited by offspring from their parents. This can cause some alleles not to be passed down at all, leaving others "fixed" as the only alleles present at the locus

allelic frequencies

The unusually cyclic structure of proline creates a high degree of rigidity in the primary structure of a protein and disrupts the normal formation of _____ structures. Thus proline is most likely to be found in the "unstructured" turn regions of a protein between the larger secondary structures

alpha and beta secondary

Glucose polymer in liver (glycogen) is formed by glycosidic bonds between glucose molecules through _____ linkage linearly and ____ linkage at branch point

alpha(1-4), alpha (1-6)

An _____ enzyme can remove the amino group from a residue and replace it with carboxylic acid or carbonyl functionality

aminotransferase

____ means "able to act as an acid or base"

amphoteric

Proto-oncogenes do not necessarily exhibit a greater or lesser rate of transcription in cancer patients. It is the activated form of a proto-oncogene, ____, that relates closely to tumor growth and metastasis

an oncogene

_____ are those structures that evolved independently to carry out the same function. Thus, the wing of a bee and the wing of a bird are analogous structures

analogous structures

____ is the inappropriate development of new red blood vessels; this process is an important aspect in the development of cancer

angiogenesis

___ bind to anion exchange chromatography

anionic peptides

A bacterium has a faulty lac operon in which there is a structural defect in the operator. In this bacterium:

answer- there is a structural problem with a segment of DNA that binds to the repressor -in the lac operon, the operator is the segment of DNA that binds to the repressor. In the absence of a functional repressor/operator binding, the cell will constitutively produce the proteins needed for lactose metabolism. Although this may not be fatal to a cell, it will waste energy if the surroundings lack lactose

Uncoupling agents, such as 2,4-dinitrophenol, operate by facilitating proton diffusion across the mitochondrial membrane, dissipating the proton gradient. After administration of a concentrated solution of 2,4 dinitrophenol to an actively respiring muscle cell, the overall rate of ATP synthesis:

answer- will decrease by over 50% without immediately halting the electron transport chain uncoupling agents dissipate the proton gradient that is generated by the 4 complexes in the electron transport chain. The effect is analogous to poking holes in the inner membrane W/out the proton gradient, no flow of protons can occur through the ATP synthase to regenerate ATP Since most ATP in typical cells is produced via the action of ATP synthase during oxidative phosphorylation, ATP production will decrease by over 50% However the ETC will continue; it simply will no longer be coupled to ATP production

An opiate overdose slows the respiratory drive, resulting in decreased gas exchange across the alveoli. This can damage tissue or cause death from:

answer-cell damage resulting from reduced ATP levels If the respiratory rate slows, the transport of oxygen to tissues will decrease as well. In such conditions, cells cannot generate ATP through mitochondrial respiration, though they can still use anaerobic processes such as glycolysis and fermentation. Therefore, opiate overdose leads to decrease in ATP concentration in the cells of the vital organs, inhibiting their function

to make ____, mRNA is isolated, reverse transcribed and amplified using PCR

cDNA

____ are transmembrane proteins which play a primary role in cell-to cell adhesion, forming adherens junctions to bind cells within tissues together

cadherins

The parathyroid glands produce and secrete parathyroid hormone (PTH) which opposes ____ to regulate calcium levels. PTH increases plasma calcium by increasing Ca 2+ absorption in the small intestine and extraction of the ion from the bone. In other words, high PTH results in high plasma calcium and low bone density.

calcitonin

____ decreases plasma concentrations Ca 2+ by exerting a number of effects. One key example is that calcitonin stimulates bone formation by increasing osteoblast activity and decreasing osteoclast activity. The antagonist hormone to calcitonin is parathyroid hormone (PTH) which acts to increase plasma Ca 2+ concentrations by decreasing bone formation

calcitonin

____ stimulates osteoblasts decreasing calcium serum concentration

calcitonin

The combination of CO2 and H2O makes ____. Carbonic acid and its conjugate base (bicarbonate) play a large role in maintaining pH of the human body

carbonic acid (H2CO3)

Carboxylases are enzymes that catalyze ____ or ____ reactions

carboxylation; decarboxylation

Only the heart itself contains ____ muscle

cardiac

Injuries to cartilaginous tissue, such as torn and stretched ligaments, can often take quite some time to heal. Which statement best explains why injuries to this type of connective tissue heal so slowly?

cartilage is not vascularized, and therefore cannot easily obtain the nutrients to rebuild damaged tissue. Instead it must rely on diffusion from the surrounding ECM

Glycolysis is a ___ process with a negative net change in free energy

catabolic

____ is measured by Kcat/Km, which means that a higher Km and a lower Kcat gives the lowest efficiency

catalytic efficiency

____ occurs primarily through different gene expression levels

cell differentiation

Tumor suppressor genes, also known as antioncogenes, regulate ___

cell division

Oncogenes are genes that are involved in ___ and have the potential to cause cancer

cell growth

____ involves the activation of phagocytes, antigen specific cytotoxic T lymphocytes, and the release of various cytokines in response to antigen

cell mediated immunity

A _____ is composed of short lengths of microtubules arranged in the form of an open-ended cylinder, designed to help chromosomes separate during cell division

centriole

Pepsinogen is a zymogen, or inactive enzyme precursor, that is released by ____. This prevents pepsin (the active form and also a protease) from degrading stomach cells while it is stored.

chief cells

_____ are catecholamine-secreting neuroendocrine cells of the adrenal medulla. Catecholamines include epinephrine and norepinephrine

chromaffin cells

___ catalyzes a irreversible reaction in Kreb's cycle

citrate synthase

What is the function of ribonuclease?

cleaves RNA

mRNA and hnRNA are examples of RNA that do ____

code for proteins

The nucleotide sequence of both the ___ and ___ are complementary to the sequence of the template strand

coding strand and mRNA

multiple ___ occur for the same ___

codons; amino acid

A ____ is a nonprotein compound that is required for activity of a protein; we should know that vitamins and other molecules are often cofactors that help enzymes drive chemical reactions forward in biological systems; cofactors are "helper" enzymes

cofactor

___ is a primary component in connective tissue such as that found in the tendons, cartilage, and blood vessels

collagen

_____ contributes heavily to the tensile strength of connective tissue in the skin. It is a fiber in the extracellular matrix

collagen

______ is an analytical method used to purify or prepare a compound of interest from a mixture of other compounds. Columns often separate on the basis of charge, size, or the polarity of a target compound

column chromatography

snRNAs

combine with proteins to form splicesome which splices introns out of the pre-mRNA transcript. The exons can than be rejoined to form the mature mRNA

The binding affinity of hemoglobin for carbon monoxide (CO) is over 250 times greater than its affinity for oxygen. Specifically, CO is a ___ that binds hemoglobin directly at the heme site

competitive inhibitor

NADH dehydrogenase is also called ____

complex 1

As electron carriers, NADH and FADH2 must be oxidized to generate the proton gradient used for ATP synthesis. NADH is oxidized by ____ while FADH2 is oxidized a step later by ____

complex 1; complex 2

Cytochrome C, also known as ____ is an intermediate in the ETC

complex 3

_____ changes can expose new interfaces of an enzyme. This method too can be used to convert a zymogen into its enzymatic partner

conformational

____ is a feature of gram negative bacteria

conjugation

bacterial ____ is the transfer of genetic material, ordinarily in the form of a plasmid, between bacterial cells by direct cell-to-cell contact or via a bridge-like connection between the two cells.

conjugation

Capillary walls consist only of a single endothelial layer. Since they lack smooth muscle entirely, they cannot possible ____ or ___

constrict or expand

Myoglobin has only one oxygen binding site. Consequently it does not bind oxygen cooperatively. Hemoglobin, in contrast, utilizes _____

cooperative binding

The unfolding of a protein is a _____. Cooperative processes are marked by sigmoidal curve

cooperative process

When you have an enzyme with a hill coefficient greater than 1, this means the enzyme displays _____

cooperatively

Telomerase

creates short, repetitive sequences known as telomeres at the ends of chromosomes. These regions of DNA serve to protect the chromosome from general breakdown in the nucleus and from the loss of genetic information during replication

____ is a 3 carbon molecule able to form disulfide bonds with other cysteine molecules

cysteine

Complex lll transfers electrons from coenzyme Q to _____

cytochrome C

Helper T cells also produce ____ that fully activate macrophages, allowing them to establish a more acidic phagosomal pH and kill pathogens more effectively

cytokines

Glycolysis, Gluconeogenesis, and Fermentation all take place in the _____

cytosol

____ use perforin and granzymes to lyse target cells

cytotoxic T cells

Tom is conducting a biochemistry experiment. He knows that a certain enzyme is present in his test tube, along with unknown concentrations of substrates and a competitive inhibitor. After addition of a quantity of substrate, Tom might observe that: a)the reaction rate remains the same b)the reaction rate increases linearly c)the reaction rate increases nonlinearly d)any of the above could occur

d)at a low substrate concentration, addition of more substrates will cause the rate to increase linearly when vmax is approached (at high substrate concentration) the rate will increase nonlinearly After Vmax is achieved, the addition of substrate will not impact the rate at all

Although PCR protocols almost exclusively employ heat to induce DNA denaturation, alternative denaturation methods are often used for Southern Blotting. Which of these solutions would be least effective when separating DNA strands to enable adherence to the blotting membrane? a)a mild sodium hydroxide b)6 M urea solution c)sodium dodecyl sulfate solution d)a mild Mg2+ solution

d- magnesium has a well known stabilizing effect on DNA. This can be inferred from the essential role of magnesium as a cofactor for DNA polymerase. In this role Mg 2+ ion coordinates with the pyrophosphate leaving group, stabilizing its transition into solution c- would be an unconventional choice but any amphipathic molecule has the potential to disrupt hydrophobic interactions

uracil can be formed by the direct ____ of cytosine while thymine is formed from a deamination and ____

deamination; methylation

Insulin acts to ____ plasma glucose concentrations

decrease

2-3 BPG binds hemoglobin and causes a ____ in oxygen affinity. Fetal insensitivity to 2-3 BPG permits heightened ____ of fetal hemoglobin. In this way, its analogous to myoglobin

decrease; saturation

Expansion of the thoracic cavity leads to ____ in the pleural cavity, resulting in subsequent increase in lung volume

decreased pressure

Polypeptides are formed via _____ reactions that link amino acids with peptide bonds. The incoming amino group acts as a nucleophile and attacks the carbonyl carbon

dehydration synthesis

____ involves the removal of hydrogen to form a double bond

dehydrogenation reaction

____ refers to any phase in which the cell becomes more positive (or less polarized) than its resting membrane potential

depolarization

The ___ is located immediately above the hypodermis. Technically, it is part of the cutaneous layer and is composed of connective tissue

dermis

SDS is a ____ used to ensure that polypeptides have a negative charge proportional to the length of the molecule. The phosphate backbone on nucleic acids provide a negative charge on the molecule . This allows the nucleic acids to migrate towards the positively charged anode, with the gel acting as a sieve to separate sequences based on size

detergent

Fraternal twins are the result of the fertilization of two ___. Thus fraternal twins share 50% of their genes making them no more closely genetically related than any two siblings

different ova by two different sperm

Aldosterone is released from the adrenal cortex in response to low blood pressure. Its primary function is to increase sodium reabsorption in the ___ and ___. Aldosterone up regulates the sodium-potassium pump along the lining of the nephron, pumping 3 sodium ions out of the nephron lining toward the blood for every 2 potassium ions it pumps in toward the nephron away from the blood. Since we have a net solute movement out of the nephron, aldosterone also increases the gradient that favors water reabsorption

distal tubule and collecting duct.

Quaternary structure is stabilized by

disulfide bonds and other non-covalent interactions

_____ are those that form from two free S-H bonds yielding S-S bonds. To break these linkages, one must simply add hydrogen back to the two sulfur atoms involved. A reducing agent accomplishes this goal.

disulfide linkages

Resistance to a particular strain of pathogen ____inherently confer resistance to other strains

does not

Integrase is an enzyme responsible for integrating viral ____ into the host genome

dsDNA

Both bile and lipase are released into the ___ to act

duodenum

____ is a motor protein that moves cellular "cargo" along microtubule "train tracks"

dynein

Prokaryotic translation relies on several release factors. In contrast, eukaryotes need only one such factor; this factor being called ___

eRF1 (eukaryotic translation termination factor 1)

The _____ technique is used to analyze protein post-translational modifications, such as lipids, phosphoro-moieties and glyco-conjugates. Eastern blotting can be considered an extension of the biochemical technique of western blotting

eastern blot

The spinal cord as well as the rest of the central nervous system arises from the ____ tissue

ectodermal

Directional selection favors ____

either of the extremes, never both

____ gives connective tissues elasticity

elastin

NADH is produced during beta-oxidation and is then used as an electron carrier for the ______

electron transport chain

Pinocytosis is a specific form of ____. As such it requires large amounts of ATP and is certainly characterized as active transport. Phagocytosis is another special form of endocytosis

endocytosis

The ___ gives rise to the lining of several internal organs

endoderm

The lining of the respiratory tract and bladder are derived from the ___

endoderm

Secreted proteins are cleaved into mature form within the _____

endomembrane system

______ is highly specific enzymes which recognize and hydrolyze phosphodiester linkages at unique palindromic sequences in DNA

endonucleases

The Golgi apparatus functions like a cellular "post office". It receives proteins from the ____ and packages them into vesicles. These membrane-bound sacs can then travel to specific locations within the cell or to the plasma membrane for excretion. Additionally, lysosomes are formed from vesicles that bud off the Golgi's larger structure.

endoplasmic reticulum

During endocytosis, external molecules or pathogens are first engulfed in an invagination of the cell membrane known as a vesicle. These vesicles initially deliver their contents to early ____, which are also membrane bound. These contents then progress to late endosomes, which fuse with lysosome for degradation.

endosomes

The most popular theory for the origin of the mitochondria is known as the ______. This theory states that these organelles began as small, independent prokaryotic organisms that entered into a symbiotic relationship with a eukaryotic cell. Additionally, mitochondria have a separate set of DNA that unlike nuclear DNA is inherited maternally. They are also able to replicate outside of cell replication

endosymbiotic hypothesis

The blood brain barrier is composed of ______ that prevent the movement of most solutes

endothelial cells with tight junctions

The ____relates to the control of the digestive organs

enteric nervous system

____ depends principally on an enzyme's intrinsic catalytic efficiency, its concentration, the initial substrate concentration, the presence of inhibitors or allosteric activators, temperature and pH

enzyme activity

In the CNS, the ___ cells do use cilia to circulate CSF throughout the brain and spinal cord. This function is essential for the effective clearing of extracellular waste

ependymal

The most exterior layer of the skin is called the ___

epidermis

ectodermal cells differentiate into "external" features such as ____, ____, ___ or ___. It also gives rise to the nervous system via neurulation

epidermis, lens of eye, hair or nails

ovarian cells are ____ and osteoclasts are ____

epithelial cells; connective tissue cells

The presence of a single large band on SDS page could be consistent either with the presence of multiple bands with _____ or the protein is composed of a single subunit

equal masses

___ are anucleate

erythrocytes

___ does inhibit bone resorption; in fact, this is the reason why post menopausal women experience significant declines in bone density

estrogen

___ and ___ together cause endometrium to thicken. Estrogen produced from developing follicles stimulate endometrial growth and progesterone is responsible for converting the estrogen primed endometrium into a receptive state

estrogen and progesterone

Plants and fungi contain membrane bound organelles, including nuclei, mitochondria and chloroplast. Both of these groups consist of ____ organisms

eukaryotic

Protozoa are classified as part of the kingdom Protista. Although these organisms are unicellular and simple, they are ____

eukaryotic

Myelination has no effect on ____. It only affects conduction velocity

excitability

Glutamate is a classic _____ neurotransmitter

excitatory

If more than one ____ arrives in a very short interval, the neuron cannot return to a resting potential in the time between the arrivals. Therefore, multiple inputs over a short period of time increases the likelihood of reaching threshold, which is necessary for an action potential to occur

excitatory post synaptic potential (EPSP)

____ products, generally enzymes, are secreted through ducts rather than the bloodstream.

exocrine

The liver, pancreas, and gastric chief cells are example of ___. The liver and pancreas secrete many hormones into the bloodstream, but also releases digestive enzymes into the lumen of GI tract, an exocrine function. Gastric chief cells are purely exocrine and secrete pepsinogen into the stomach

exocrine glands

A smaller protein subunit travels ___ along the SDS-page whereas a large protein subunit travels a ___ distance.

farther; smaller

______ are low in myoglobin and whitish in appearance. These cells depend heavily on anaerobic respiration; for this reason they require relatively little oxygen

fast glycolytic fibers

_____ do contain abundant myoglobin; however they contract rapidly and while resistant to fatigue are less so then slow oxidative fibers

fast oxidative fibers

____ is a mechanism by which the end products of a pathway may inhibit one of the enzymes involved in an earlier step. In this way, production of the product is slowed

feedback inhibition

___ is an iron-sulfur protein that is the final electron acceptor in photosynthesis

ferredoxin

___ is embedded in the extracellular matrix. These are the cells that produce the fibers comprising connective tissue such as collagen

fibroblast

____ refers to the general movement of plasma from the glomerulus (a tight network of capillaries) into the entrance of the nephron at the Bowman's Capsule. This process is extremely general, with virtually any molecule smaller than protein allowed passage into the filtrate

filtration

Chymotrypsin is an enzyme that selectively cleaves peptide bonds at the carbonyl end of the aromatic amino acids. A peptide sequence is given below: LYPPGLWNKYATSYF How many potential chymotrypsin cleavage sites are located in the structure of this molecule?

first off recall the three aromatic amino acids are tryptophan (W), tyrosine (Y), and phenylalanine (F). The peptide sequence is written in the N to C direction. So we can ignore the aromatic amino acid at the C termini, making there 4 cleavage sites.

____ is a main component of bacterial flagella

flagellin

The sternum is an example of a ____ bone

flat

sorting, identifying, and counting individually labeled cells is accomplished via ____

flow cytometry

Unsaturated fats give cell membranes greater ___ and functionality, while lowering the freezing temperature of the membrane as a whole

fluidity

Surface residues are less likely to be "keystones" for the protein structure, as those sorts of residues tend to be found in the protein's core. A surface residue that isn't critical for structure may still be critical for the _____ of the protein and may be important for binding substrate

function

connexin is a ___. Cadherins form ____. Occuldin comprises ____

gap junction; cell to cell junctions; tight cell to cell junctions

____is the stage in embryonic development in which the single-layered blastula becomes the 3 layered gastrula , the 3 germ layers of which are the ectoderm, endoderm and mesoderm

gastrulation

_____ is a method for separating nucleic acids and proteins (or fragments of these molecules) on the basis of size and/or charge.

gel electrophoresis

Repressors inhibit ___ by blocking specific regions of the DNA or by preventing RNA polymerase from attaching to the promotor

gene expression

Transmembrane proteins such as G-protein coupled receptors bind hormones and other ligands to activate signal transduction pathways. This activation ultimately results in a change in ______. Transmembrane proteins can form channels that allow charged proteins to pass. ATP synthase spans the inner mitochondrial membrane and is therefore a transmembrane protein

gene expression

"dominant" refers to interaction between ___

genes, not which trait is simply most common

transduction involves the transfer of ____ through a viral vector

genetic material (generally DNA)

The ____ is the ball of capillaries from which filtrate initially enters the Bowman's Capsule

glomerulus

Pancreatic alpha cells release ______, a peptide hormone that increases blood glucose levels

glucagon

____ is released by the pancreas as a response to low blood glucose levels. Its main purpose is to increase glycogenolysis to increase blood glucose. Most of glucagon's actions occurs in the liver

glucagon

One goal of the fight or flight response is to increase the glucose that is available to the body for use. To do so, epinephrine triggers an increase in _____ which in turn promotes an increase in blood sugar

gluconeogenesis

____ is the pathway for the synthesis of glucose from other metabolic compounds and is thus activated when the body's stores of carbohydrates are low

gluconeogenesis

_____ catalyzes the final step of gluconeogenesis and glycogenolysis

glucose 6 phosphatase

Gluconeogenesis must overcome the 3 irreversible steps that occur during glycolysis. These enzymes used to achieve this are __, ___ ,____and ___

glucose 6 phosphatase, fructose 1,6- bisphosphatase, PEP carboxykinase and pyruvate carboxylase

Production of PEP, glucose, and fructose-6 phosphate by gluconeogenesis-specific enzymes that bypass irreversible steps of glycolysis push the equilibrium of reversible enzymes that function in both glycolysis and gluconeogenesis in the direction of _____

glucose production

Glycolysis is the anaerobic breakdown of ___ into ____

glucose; pyruvate

___ is the most common excitatory neurotransmitter and stimulates excitatory receptors

glutamate

Glycognenesis is stimulated during times of energetic abundance, allowing excess glucose to be stored as ___ for later use

glycogen

Which of these enzymes manipulate alpha-1, 4 linkages in polysaccharides l. glycogen synthase ll. glycogen branching enzyme lll. glycosyltransferase lV. glycogen phosphorylase

glycogen synthase adds glucose units to a growing chain by forming novel alpha-1,4 linkages A glycosyltransferase enzyme removes a three sugar segment from a branched chain and reconnects it to the parent strand, replacing one alpha-1,4 linkage with another glycogen phosphorylase cleaves seven unit segments from chain that are branched with alpha 1,4 connections

Insulin is released in response to high blood glucose levels. It then stimulates ____ to decrease these levels and promote the generation of useful heat. Insulin also stimulates glycogenesis (the production of glycogen polymers to store the glucose for future use). To avoid a futile cycle, it simultaneously decreases the rate of gluconeogenesis

glycolysis

____ unlike the Krebs cycle and oxidative phosphorylation occurs in the cytoplasm

glycolysis

When a protein is translated by an ER-bound ribosome, it enters ER for processing, then sent to the ___ where it is packaged for export through the plasma membrane

golgi

____ modifies proteins

golgi apparatus

Typically, estrogen and progesterone negatively feedback on ______ . As a result high levels of these steroids lead to reduced release of LH and FSH. As a surge of LH is required to stimulate ovulation, persistently high progesterone levels near the beginning of the menstrual cycle will prevent the woman from ovulating--this is how birth control works.

gonadotropin-releasing hormone (GnRH)

____ are follicular cells closely associated with the developing female ooocyte (egg). They function to convert thecal androgens to estradiol prior to ovulation. After ovulation, they give rise to the corpus luteum and begin producing high levels of progesterone

granulosa cells

A Hill coefficient ____ indicates positive cooperativity

greater than 1

The lateral plate mesoderm develops into the ____ and ___. The paraxial mesoderm develops into the ___ and ___

gut and circulatory system; skeletal system and cartilage

___ is required to permit embryonic implantation and continued embryonic growth

hCG

Parallel evolution

happens when closely related species evolve in a similar manner

Centromeres are ____that assist in the connection of sister chromatids

heterochromatin-based regions of DNA

Meiosis 1 is a ____ or reductional, division. In other words, parent cells begin as diploid, while daughter cells formed during meiosis 1 are haploid

heterotypic

Glycolysis has 3 irreversible steps; these are catalyzed by ____,____, and ___. Note that the first irreversible reaction (promoted by phosphofructokinase) is often termed the committed step

hexokinase, phosphofructokinase, and pyruvate kinase

Myoglobin has a ___ affinity than hemoglobin allowing it to draw oxygen out of the blood effectively

higher

A _____ is an enzyme attached to a cofactor

holoenzyme

_____ is found to repair DNA damage in eukaryotes and is especially useful in repairing double stranded breaks

homologous recombination

_____ are those that have similar evolutionary history, arising from the same source, even if they now have different functions. The forelimbs of mammals (human arms, walrus flipper, bat wings) would all be homologous despite their different functions

homologous structures

Meiosis ll is a ___ or equational, division.

homotypic

The endocrine system uses ___, which are signaling molecules that travel via the circulatory system. As such, they can move throughout virtually the entire body and generally act on distant targets

hormones

Helper T cells are critical for the activation of the _____ (B cells) and the ____(T cells)

humoral immune response and cellular immune response

During synthesis of malate, fumarate experiences the addition of water across its double bond. This results in the replacement of this bond with an -OH group on one carbon and a hydrogen atom on another. Such a process is a classic example of ____. The opposite process where a loss of water occurs is called ___

hydration; dehydration

Gastrin is secreted by G cells and promote the release of _____ by parietal cells in the stomach

hydrochloric acid

A particular protein largely lacks both secondary and tertiary structure. Which factor, if any, is mainly responsible for the resting state of the protein? a)entropy b) hydrogen bonds between amino acid residues c)dipole-dipole interactions d)none of the above

hydrogen bonds, disulfide bonds, and dipole-dipole interactions produce the complex folding patterns seen in secondary and tertiary structure. However, if a protein does not have these elements of structure, it will adopt a state in which entropy is maximized. This will result in a shape somewhere between a state of tight, complex folding and one of completely linear arrangement answer-entropy

B-galactosidase catalyzes the cleavage of lactose into B-galactose and glucose in a classic ____ reaction

hydrolysis

___ requires the use of water to break bonds

hydrolysis

_____ reactions are a very common way to activate zymogen. Water removes a leaving group, freeing the catalyst in its active form

hydrolysis

Proteolysis is the breakdown of proteins into smaller polypeptides or amino acids via ____

hydrolysis reactions

water is used to promote ____ which breaks bonds

hydrolysis reactions

Lipase _____ fatty acids

hydrolyze

Parathyroid hormone increases blood calcium concentrations by activating osteoclasts; these cells reabsorb _____, a calcium containing mineral present in bone

hydroxyapetite

Michealis Menten kinetics describes a _____ dependence of Vo on substrate concentration

hyperbolic

type 2 diabetes is characterized by ____ due to insulin resistance (ex: cells do not respond properly to insulin)

hyperglycemia

with regard to the skin, another name for the subcutaneous layer is?

hypodermis The hypodermis is the most interior layer of the skin and consists mostly of fat deposits

Antidiuretic hormone is produced by the ____and stored and released from the posterior pituitary

hypothalamus

The ____ releases Cortisol releasing hormone (CRH) which stimulates the ____ to release adrenocorticotropic hormone (ACTH) which stimulates the ____ to release cortisol

hypothalamus, anterior pituitary, adrenal cortex

____means weakened immune system

immunocomprimse

____ uses antibodies that are specific to proteins in an attempt to determine which antigens are expressed in a particular region of tissue. This technique has particular applications to cancer research

immunohistochemistry

"blending" is a classic sign of

incomplete dominance

Competitive inhibition causes the apparent km of an enzyme to ____while the vmax remains constant

increase

Unsaturated fatty acids tend to _____ the fluidity of membranes

increase

Oncogenes are activated proto-oncogenes that tend to result in ____

increased cell proliferation

Mendel's Law of ______ states that separate genes which encode separate traits are passed down from parent to offspring independently of each other. This is because the genetic alleles on chromosomes are sorted independently due to separation of homologous chromosomes during the meiosis stage of the reproductive process

independent assortment

For CAP to work, the repressor must have also been removed by an ___

inducer (allolactose in the case of a lac operon)

movement of leukocytes into mucosal tissue tends to cause _____

inflammation

Oncotic pressure is simply another term for the osmotic pressure that exists due to proteins. This phenomenon is necessary to promote the ____ into the distal regions of the capillary bed. Basically, albumin (like other proteins) is too large to diffuse in and out of a capillary bed. Instead this protein stays in the capillary bed and draws water, with its associated toxins and waste products, from tissues to the bloodstream

influx of water

Glucose-6 phosphate is the product of Step 1. Thus when it is presented in excess, it will ____

inhibit the forward reaction of this step through a classic negative feedback mechanism

In a negative inducible operon, transcription is ____ with a repressor. This results in a basal transcription rate near 0. However, transcription can be "switched on" by the addition of an ____ which blocks the repressor from binding to the operon

inhibited; inducer protein

GABA is a classic ____ neurotransmitter

inhibitory

_______ will generally hyperpolarize a neuron or otherwise inhibit its firing

inhibitory post-synaptic potential (IPSP)

Cyclins and Cdks are crucial cell cycle regulators. Depending on the phase of the cycle, certain Cdks will be expressed at higher concentrations than others; this signals the cell to prepare for the next transition. In other words, the levels of these proteins are dynamic throughout the cell. If the cell has perpetually low concentrations of cyclin and Cdks, it lacks the signaling molecules to ____

initiate and prep the cell for upcoming phases

The ____ is the mass of cells within the blastocyst that will eventually give rise to the fetus

inner cell mass

The electron transport chain takes place across the _____

inner mitochondrial membrane

Silver Chloride (AgCl) is an example of an ____ compound

insoluble

During ____, contraction of the diaphragm and internal intercostal muscles lead to expansion of the thoracic cavity and a decrease in intra-pleural pressure. This negative pressure, relative to the atmospheric pressure at the entry of the upper airway, generates airflow through respiratory tree and to its terminal extension--the alveoli

inspiration

____ are transmembrane receptors that modulate cell-to-extracellular matrix interactions. Specifically, these proteins often attach the cell to collagen and fibronectin fibers

integrins

The electron transport chain uses the free energy from redox reactions to pump proteins from mitochondrial matrix to ____thereby generating an electrochemical gradient across the inner mitochondrial membrane

inter membrane space

____ such as keratin are directly responsible for structural integrity of the skin

intermediate filaments

Commensalism

involves a benefit to only one participating species; the other is unaffected

Heme is the ____ cofactor that allows hemoglobin to carry oxygen

iron-binding

A step in a reaction with an extremely negative Gibbs free energy means this step is ____ or is called the "committed" step

irreversible

ectosymbiosis

is a specific phenomenon in which one species lives on the surface of another

At equilibrium, change of G=0. With no free energy change, the system ____

is unable to do work

____ is used to separate proteins by isoelectric point, which depends on the amino acid composition of the protein

isoelectric focuing

Isoelectric focusing separates proteins based on an _____

isoelectric point (the pH at which the net charge of the protein is 0)

Different ____ of proteins are expressed from single genes through alternative splicing of exons of the primary transcript

isoforms

Allosteric inhibition involves the binding of molecules at a site other than the active site. The binding causes conformational change in the enzyme rendering _____

it unable to bind its original substrates

____ is used to describe the rate limiting step of catalysis under saturating conditions of substrate

kcat

catalytic efficiency of an enzyme is measured by ___ ratio

kcat/Km

____, a structural protein found in large amounts in the epidermis, gives the skin its flexibility and toughness

keratin

_____ extend across epithelial cells to link adjacent cells via structures called desmosomes. As a type of intermediate filament, keratin possesses high tensile strength; as such, it can form a network of fibers that distribute mechanical stress among the cells of an epithelial layer. Keratin is therefore the the cytoskeletal protein that contributes most to the skin's stretching ability

keratin fibers

____ are made as a last resort when energy sources are insufficient

ketone bodies

In humans, gluconeogenesis occurs in the ___. Gluconeogenesis mainly takes place in the liver but does occur to a lesser extent in the kidneys

kidneys

____are a class of enzymes that are responsible for the addition of phosphate groups. The fact that phosphorylation is reversible is advantageous especially in regulatory proteins that must be turned on and off. Phosphate groups may be attached to amino acids that have a terminal oxygen atom in their R group, like tyrosine

kinases

___ is the substrate concentration required for a reaction to reach 1/2 vmax

km

The ribose ring of RNA will be heavier than the deoxyribose of DNA because DNA ____

lacks a 2'-OH which is present on the ribose of RNA

In the term "fluid mosiac model" of a plasma membrane, the fluid aspect refers to the ability of component molecules to travel _____ within the leaflet of the bilayer. While the mosiac term denotes the presence of ____ and other molecules that are scattered in a mosiac within the structure

laterally; proteins

eukaryotic DNA polymerases are responsible for the elongation ____ and ____ during _____

leading and lagging strand; DNA replication

A mutation in the promotor region will make the initiation of transcription ___likely

less

____ are metabolically active and produce many proteins to fight pathogens and digest phagocytosed material

leukocytes

Organisms can obtain energy either from ___ or organic or inorganic electron-containing compounds ___

light (photo); chemo

As a steroid hormone, for ex: testosterone is derived from cholesterol and has significant non-polar character. Thus it can easily pass through the relatively non-polar _____

lipid bilayer

Where in the eukaryotic cell would you be most likely to find the highest concentration of lipid peroxides? a)cell membrane b)nuclear membrane c)inner mitochondrial membrane d)golgi apparatus

lipid peroxides are formed by free radical attack on a lipid. Thus we expect the location with the most lipid peroxidation to be someplace where oxygen is participating in a redox reaction. It happens that electron transport chain which is located on the inner mitochondrial membrane, has oxygen as its terminal electron acceptor

Lipase is used to digest ____ in the small intestine and is also known as pancreatic lipase, a term that denotes the organ in which it is produced

lipids

Insulin is an inhibitor of ____an enzyme responsible for hydrolysis of triacylglerides to free fatty acids

lipolysis

The femur is an example of a ___ bone

long

The ____ prepares the medulla of the kidney for later absorption of water

loop of henle

In biochemistry, the dissociation constant (K) is useful to quickly compare the binding affinities of enzymes and substrates. A ___ K value means the ES complex does not readily dissociate into enzyme and substrate (in other words, it is relatively stable). The lower the K value the greater the enzyme-substrate binding affinity

low

DNA nucleotides have ____ molecular weights than RNA nucleotides due to the missing 2' OH

lower

_____ is a hormone produced by gonadotropic cells in the anterior pituitary gland. In females, an acute rise in LH triggers ovulation and development of the corpus luteum. In males, LH stimulates Leydig cells to produce testosterone

luetenizing hormone

The _____ exchanges fluids between the tissues and the circulatory system

lymphatic system

During ____ phase, the virus is dormant and its DNA is integrated with that of the host cell genome. All of the daughter cells from the initial infected cell will carry the virus's genetic information. At some point, the virus will be and will hijak the cellular machinery in order to produce viral progeny. This marks the entry into the lytic cycle

lysogenic

The _____ occurs when the phage genome is incorporated into the host genome, allowing it to be replicated as the host cell replicates. In this phase there is no active infection. Viruses able to transition between lysogenic and lytic cycle based on environmental and cellular factors, such as stress.

lysogenic cycle

The ___ is the organelle in the cytoplasm of eukaryotic cells containing degradative enzymes enclosed in a membrane

lysosome

____ digest food and waste material

lysosome

____ facilitate the enzymatic catabolism of protein

lysosomes

____ function to digest toxins, proteins, and unusable cellular macromolecules. These organelles are membrane-bound and contain enzymes that function at low pH.

lysosomes

During the ____ phase, the host cells are actively producing more virus progeny and are being lysed so that the virus can go and infect other cells

lytic

In the ______, host cell machinery is taken over to create more phage proteins, causing cell death (lysis).

lytic cycle

viruses have two life cycle: ____ and ____

lytic cycle and lysogenic cycle

Northern Blot asses for ___

mRNA

Northern Blotting is best used for detecting and identifying _____

mRNA and rRNA

The sense strand has the same same sequence as the product ___, except sense DNA has a thymine instead of uracil

mRNA that is transcribed

____ found in the blood are derived from monocytes that originate in the bone marrow

macrophages

A multifactorial trait is dependent on ____

many genes

dendrocytes are bone-marrow derived cells abundant in normal human body including the dermis. They occupy the perivascular space where they are closely associated with ____

mast cells

Mitochondria is a classic example of ____ inheritance

maternal

Virtually all of the cellular machinery and organelles are ____ inherited. Paternal organelles are destroyed during fertilization

maternally

The citric acid cycle occurs in the ____

matrix of the mitochondria

A ____ consists of a 5' cap (one base pair), 5' UTR, coding regions, 3'-UTR, and a 3' poly A tail

mature mRNA

At equilibrium, there are no energy gradients within the isolated system, so entropy is ___

maximized

glomerular filtrate is most concentrated in the _____ of the collecting duct

medullary portion

___ is used to generate sex cells (gametes), not somatic cells

meiosis

Crossing over occurs only during ____

meiosis l

___ a pigment derived from tyrosine, provides color to the eyes and the skin

melanin

The ___ gives rise to the musculo-skeletal system, circulatory system, and kidneys

mesoderm

muscle, cardiovascular system, and skeletal system are classic ____ tissue

mesodermal

The result of a saponification reaction is the production of glycerol and fatty acid. These structures are assembled into small spheres known as ____ in the lumen of the small intestines. Finally these micelles are converted into chylomicrons inside intestinal cells.

micelles

____ perform maintenance in the brain, while ___help support the brain-blood barrier

microglia; astrocytes

_______, as the name implies are used to bind bundles of microtubules together. This is most notably observed in eukaryotic flagella.

microtubule linking proteins

Tubulin is the protein component of _____; specifically, dimers of alpha and beta tubulin polymerize to form microtubule fibers

microtubules

_____ are involved in mitosis, meiosis, cell trafficking of vessels, and ciliar/flagellar motion. Microtubules are composed of tubulin protein dimers.

microtubules

Cytoskeleton: ____ form the mitotic spindle and maintain cell shape ____ is the microtubule-organizing center _____ is fibrous proteins that hold organelles in place ____ is fibrous proteins; from the cellular cortex

microtubules centrosome intermediate filaments microfilaments

____ and ____ are cytoskeletal components used largely to give cell motility and to transport substances inside the cell

microtubules and microfilaments

____ produce energy

mitochondria

Gluconeogenesis take place in the ___ and ___

mitochondria and cytoplasm

Certain tissues in the body are able to regenerate due to the ability of their cells to undergo ____ when necessary

mitosis

small nucleolar RNA (snoRNA) are involved in

modification of rRNA and as such are located in the nucleolus where ribosomes are assembled

Because each gene has its own transcription initiation site in eukaryotes, this makes for ____ mRNA (one gene per transcript), unlike prokaryotic mRNA

monocistronic

mRNA molecules are said to be ____ when they contain coding sequences from a single polypeptide. this is the case for most eukaryotic mRNAs

monocistronic

A good way to remember the development of the zygote is using the mnemonic "more blasting gas , I'm nervous" outlines the process of _____

morula to blastula to gastrula to neurulation

Adult stem cells are ___, so there development is more restricted. Multipotent stem cells can still develop into more than one cell type but not to the extent of pluripotent cells

multipotent

Myoglobin is found in the ____while hemoglobin is found only in the ____

muscle; blood

____ is a substance that holds oxygen in the muscles and organs

myoglobin

when a virus is heat shocked, it is rendered incapable of infecting host cells. Therefore we would expect the cells incubated with heat shocked phage to not form plaques, a sign of viral infection. Controls in which we expect to see negative result (no result) are called _____

negative controls

____ is apparent decrease in affinity once a ligand molecule has bound

negative cooperativity

inflation of lungs in mammals is accomplished by _____ pumping action

negative pressure

Ectodermal tissues is found on the outer layer of the gastrula and develops into the surface ectoderm, ___ and ___

neural crest and neural tube

The ___ develops into the CNS, which includes the brain and spinal cord

neural tube

Hydrolysis of ATP into ADP and inorganic phosphate restores the myosin head to a 90 degree angle, "cocking" it for _____

new attachment and the ensuing power stroke

________ is that which is never translated into a peptide product. Examples include: microRNA, snoRNA (small nucleolar RNA), rRNA, and tRNA

non coding RNA

hnRNA is not a ____ because it codes for peptide products

non-coding RNA

nucleotide production is optimized in the ____ of the pentose phosphate pathway

non-oxidative phase

A ____ binds the enzyme and enzyme substrate complex with the same affinity. Vmax is affected but Km does not change

noncompetitive inhibitor

If a neomycin resistance gene was unusually susceptible to mutation, then there would be an increase in likelihood of a mutation rendering the resistance of the gene to be ___. Without that resistance, fewer colonies would be able to grow in the gentamicin solution

nonfunctional

Increased blood viscosity is a known contributor to the development of chronic hypertension. Which of the following is likely to increase blood viscosity? a)increased fluid intake b)sickle-cell anemia c)hyperglycemia d)infusion with a saline solution

normal erythrocytes are smooth, biconcave discs that pass easily through capillaries and over one another. Sickle cells are jagged and aggregate readily, which causes increased viscosity a/d- increased fluid intake (saline or otherwise) will increase fluid to cell ratio and thus decrease viscocity hyperglycemia will cause fluid retention which will also lower viscosity

The ___ provides the primitive axis of the developing embryo and in vertebrates, develops the vertebral column

notochord

The chora-mesoderm develops into the ____. The intermediate mesoderm develops into the ____.

notochord; gonads and kidneys

The membrane of the endoplasmic reticulum is an extension of the outer leaflet of the ______. Its removal would greatly diminish overall nuclear integrity, likely causing the organelle to lyse. As this would almost certainly trigger apoptosis.

nuclear envelope

Nucleus: _____ is the membrane enclosing the nucleus. Protein-lined pores allow material to move in and out ____ is DNA plus associated protein ____ is condensed region where ribosomes are formed

nuclear envelope chromatin nucleolus

A ____ requires a nuclear localization domain for nuclear localization domain for nuclear translocation and a DNA binding domain for binding to regulatory regions of targeted genes. In contrast, _____ are protein domains required for proteins that are directed toward secretory pathways

nuclear protein; signal sequence domains

Nucleases are enzymes that cleave the ____

nucleic acid

A ___ is the nuclear subdomain that assembles ribosomal subunits in eukaryotic cells

nucleolus

___ is a region within the nucleus that is vital for synthesis of ribosomes from protein and rRNA

nucleolus

A ___ includes a 5 carbon sugar attached to a nitrogenous base

nucleoside

The southern blot process breaks DNA into fragments using a restriction enzyme, runs the fragments on a gel, and then uses a specific labeled sequence as a probe. The DNA sequences are spaced out on the gel by length, where distance traveled is inversely proportional to fragment length. The purpose of Southern Blotting is to look for specific _______ in the DNA being tested and/or identify the size of DNA fragments that contain the sequence of interest

nucleotide sequences

Parasitism

occurs when one organism benefits from the relationship at the expense of the other

An ____ is a protein that contributes to the survival or proliferation of cancerous cells

oncoprotein

Capillaries are the smallest blood vessels in the body, with the walls that are composed of ______ of endothelial cells.

only a single layer

One round of ___ yields one ovum and multiple small, dark polar bodies. These "extra" cells later undergo apoptosis. As a result of this phenomenon, the ovum (which is demanding of both nutrients and energy) is able to retain most of the original organelles and cytosol.

oogenesis

The leading and lagging strands are synthesized in ____ directions

opposite

____ are inhibited in times of low Ca2+; activation of these cells would build bone tissue while further reducing plasma calcium levels

osteoblasts

A decreased level of calcium in the blood will stimulate secretion of parathyroid hormone. PTH is responsible activating ____, which remove calcium from bone and increase plasma Ca 2+ concentrations

osteoclasts

Parathyroid hormone causes bone reabsorption through the activity of _____, cells which reabsorb (break down) bone to release the components back into the body

osteoclasts

PTH stimulates ___ not ____

osteoclasts; osteoblasts

____ perform bone reabsorption and recycling. When stimulated by PTH, osteoclasts facilitate the removal of calcium from the bone into circulation. In contrasts ____ are uninucleate cells that serve to create new bone tissue by secreting a mineral dense matrix

osteoclasts; osteoblasts

____ are mature osteoblasts that will eventually become surrounded by their matrix

osteocytes

Parapatric speciation occurs when segments of two distinct populations ____. Due to environmental differences, these segments may develop into two species, but individuals in the overlapping areas can typically still interbreed

overlap

Day 21 of the menstrual cycle is called _____. During this time, a follicle bursts and releases a secondary oocyte. The remainder of the follicle gives rise to the corpus luteum

ovulation

when a vertebrate oocyte divides in meiosis the cytoplasm is distributed to two daughter cells. Most of it goes to the daughter cell destined to become an ___. The other daughter cells, the polar bodies are cast off with little cytoplasm.

ovum

Dehydrogenase catalyze the _____ of substrates

oxidation

disulfide linkages are cleaved exclusively via an _____ reaction

oxidation/reduction

NADPH is produced in the ____ of the pentose phosphate pathway

oxidative phase

___ is irreversible while ___ is reversible of the pentose phosphate pathway

oxidative phase; non-oxidative phae

___ catalyzes reactions that involve an electron transfer. For example the formation of NADH (a reduced electron carrier) from NAD+ (its oxidized counterpart) indicates that the redox reaction is occurring.

oxidoreductases

Complex IV transfers electrons from coenzyme Q to the final electron accepter ____

oxygen

The generation of the proton gradient requires the successive transfer of electrons from one carrier to a more electronegative one. What is the final electron acceptor in the mammalian ETC?

oxygen which is reduced to water serves as the final electron acceptor in ETC

___perpetuates labor by increasing uterine elasticity, which leads to stimulation of uterine stretch receptors. These positively feedback to the pituitary to increase oxytocin production

oxytocin

significant ____ changes can alter the protonation state of zymogen, which can activate it. Pepsin, a protease, is formed in a similar manner in the stomach

pH

Glucagon is produced and released by the ____

pancreas

Paracrine activity involves the secretion of local signaling molecules known as ____. These factors diffuse to nearby target cells and exert a variety of effects

paracrine factors

Parathyroid hormone is stored and produced in the ____

parathyroid

Once pepsinogen is released into the stomach, the low pH denatures the pepsinogen protein to produce ___

pepsin

The stomach is a highly acidic environment where HCl causes the pH to drop below 2. Any enzymes that operate here for ex: ____, must be able to function efficiently in such conditions. In contrast _____, operates within the small intestine where pH is higher due to bicarbonate secretion (ph around 6.5)

pepsin; chymotrypsin

insulin is a _____ hormone, as is characteristic for hormones that have quick-acting effect

peptide

Insulin and glucagon, these two main pancreatic hormones, are ___

peptide based

Proteolysis is a subset of hydrolysis specific to the cleavage of _____

peptide bonds

oxytocin is a ____ hormone while progesterone is a ____ hormone

peptide; steroid

The ___ nervous system is not bathed in CSF

peripheral

____ are small membrane bound organelles that function mainly to breakdown lipids

peroxisomes

____ metabolize waste

peroxisomes

Nucleotides are linked to one another by _____ bonds between the sugar base of one nucleotide and the phosphate group of the adjacent nucleotide in a way that the 5' end bears a phosphate and the 3' end bears an OH group from the sugar

phosphodiester

______ link adjacent nucleotides in DNA

phosphodiester bonds

Step 3 of glycolysis involves the conversion of fructose 6 phosphate to fructose 1,6 bisphosphate. The forward reaction is highly exergonic and thus serves as a "committed step" and is mediated by _____

phosphofructokinase

_____ and kinases are transferase enzymes that move phosphate groups between species

phosphorylases

_____ is a prokaryotic process to reverse damage done in by UV light

photoreactivation

Allopatric speciation occurs when populations, or parts of the same population, are separated by a ____

physical barrier

Growth hormone is produced and secreted by the anterior lobe of the ____

pituitary

Hemoglobin is carried in RBCs and not found in ____

plasma

Embryonic stem cells are ____ meaning that they can give rise to virtually every cell type found in the human body

pluripotent

A typical TLC procedure includes a polar stationary phase and a nonpolar mobile phase. Thus more ___will interact with the plate and travel a shorter distance than nonpolar compounds.

polar compounds

Oxytocin, like ADH, is produced in the hypothalamus, then stored in secreted by the ____

posterior pituitary

The cerebellum receives information from sensory systems and then regulates fine motor movements. Thus, it coordinates ___,___,___ and ___

posture, balance, fine motor coordination, and speech

The synthesis of the leading strand by DNA polymerase is done continuously. The lagging strand is synthesized discontinuously via the addition of short RNA primers by the ____ activity of DNA polymerase

primase

___ is responsible for synthesizing primers

primase

DNA polymerase is incapable of starting initiation, and instead is only able to elongate existing strands. Without a ___, DNA polymerase would have no initial position to bind to

primer

zymogens and ___ are synonymous

proenzymes

Flagellin is the protein that composes _____

prokaryotic flagella

___ promotes kinks or bends in the secondary structures of proteins when incorporated

proline

Acetylation ____ gene expression. Methylation ____ gene expression

promotes; discourages

___ cleave peptide bonds within proteins

proteases

The golgi apparatus is the site of ____

protein modification

Western Blotting is used to identify _____ in a sample

proteins

Most reabsorption of biologically important solutes occur in the ___

proximal convoluted tubule

____ are heterocyclic aromatic organic compounds. They consist of pyrimidine fused ring and imidazole ring (both of with contain nitrogen)

purines

Gluconeogenesis, which occurs in the liver during periods of fasting, produces glucose from non-carbohydrate carbon substrates such as ____, ____, _____, ______, and ________

pyruvate, glycerol, lactate, TCA cycle intermediates, and the carbon skeletons of glucogenic amino acids

Sensory or afferent tracts lie towards the ____ side of the spinal cord. In contrast, motor,or efferent, tracts are positioned near the___ and lateral sides.

rear (dorsal); front (ventral)

The acceptor stem on a tRNA molecule often plays a role in the ______. tRNA molecules have acceptor stems of different lengths, and the synthetase enzymes use these distinctions to differentiate between tRNA substrates

recognition of tRNA by aminoacyl tRNA synthase

One advantage of the degeneracy of the genetic code is that it:

reduces the effect of point mutations because multiple codons may correspond to the same amino acid

Biological reductions are catalyzed by ____. For ex: addition of hydride (H-)

reductases

___ involves the addition of electron density to part of a molecule. With regard to organic reactions, reductions are typically marked by the addition of bonds to hydrogen or the removal of bonds to oxygen

reductions

The pentose phosphate pathway results in the generation of NADPH, which is utilized in ____ reactions

reductive

ETC do push protons from the mitochondrial matrix to the intermembrane space. However, that direction of motion is only seen during the establishment of the proton gradient. During the ______, H+ ions will instead flow from the intermembrane space back to the mitochondrial matrix through ATP synthase

regeneration of ATP

Reversible phosphorylation of proteins is an important _____ that occurs in both prokaryotic and eukaryotic organisms

regulatory mechanism

Voluntary muscles may contract involuntarily due to a ____. The classic example is the patellar tendon reflex, in which sudden stretching of the patellar tendon leads to an involuntary contraction of the quadriceps. Such a contraction occurs before the signal has even reached the brain

relfex arc

Prokaryotic and eukaryotic transcriptional regulation requires transcription factors, including _____.

repressor and activator proteins

Coevolution

requires that 2 species evolve in response to each other

Typed 2 diabetes is marked by high blood sugar and ____ to insulin. We would expect healthy individuals to have lower blood glucose levels overall. More importantly, these individuals would respond to insulin more rapidly

resistance

A peptide bond is an amide linkage between the amino group of one amino acid and the carboxylic acid group of another. Amides are stable because they exhibit _____ between the lone pair on the nitrogen and the double bond in the carbonyl group

resonance stabilization

_____ is the exchange of oxygen and CO2 with the enviornment

respiration

using restriction enzymes to identify polymorphisms and generate a "fingerprint" for DNA sequences is a technique known as _____

restriction fragment length polymorphism (RFLP) analysis

Dehydration involves the ____ or ____

reverse of hydration or is called hydrolysis

In ____, the stationary phase of the column is hydrophobic while the mobile phase is hydrophilic. Thus, polar molecules will interact more strongly with the polar phase and elute more quickly than nonpolar species

reverse phase chromatography

_____ catalyzes the production of DNA from an RNA template. This enzyme must have DNA polymerase activity since it builds a new DNA strand; it is also RNA-dependent since, since it reads an RNA template

reverse transcriptase

____ glycolytic steps refer to those reactions that do not serve as committed steps because they do not represent a large free energy change. In other words, the forward and reverse reactions of these processes have similar change of G values, so they lack a strong tendency to move in one direction or the other.

reversible

when the free energy change of G is equal to 0, this mean the reaction is ____. Thus at equilibrium, an enzyme can easily catalyze both the forward and reverse reactions to keep the system in flux

reversible

rRNA is a building block for the ____. Ribosomes are active during translation, but do not play a role in post-transcriptional processing. Remember these modifications occur in the nucleus while ribosomes are either free floating in the cytosol or bound to the rough ER

ribosome

Since nearly all enzymes are protein-based, it is ____ (whether free-floating or attached to rough ER) that synthesize enzymes

ribosomes

Translation or the conversion of mRNA into protein occurs on ____ in the cytoplasm or along the ____

ribosomes; endoplasmic reticulum

while we typically think enzymes are protein based (and most are) some forms of RNA do perform enzyme functions. These molecules are known as ____and play important roles in protein production and packaging.

ribozymes

After an organism dies, a condition known as ____is observed, when the muscles stiffen and the organism's limbs resist manipulation. This happens because the breaking of cross bridges between thick and thin filaments requires fresh ATP to replace ADP bound to the myosin ATPase enzyme. Without ATP, the cross bridge remains intact indefinitely

rigor mortis

The ____is dotted with round ribosomes that serve to produce a wide variety of proteins

rough ER

Cooperative binding involves binding of multiple sites on the ___ molecule not binding on entirely ___ molecules

same; different

The hydrolysis of an ester is a reaction that is sometimes known as ____. In humans,this process is catalyzed by pancreatic lipase.

saponification

The ____ law of thermodynamics states that a spontaneous reaction or cyclic process must yield a net entropy increase. In other words the entropy of the universe should be continually rising. This can be also be explained as a catabolic process that can be coupled with an anabolic process to yield a net increase in entropy

second

Spermatogonia gives rise to primary spermatocytes before meiosis even begins. Meiosis l then marks the division of a primary spermatocyte into two _____. At the end of meiosis l, which is also known as reductional division, cells are already haploid. However, chromosomes retain their identical copies in the form of attached sister chromatids

secondary daughter cells

____ or the transport of solute from the bloodstream into the nephron

secretion

coding is the ____ strand template is the ____ strand

sense antisense

SDS-page is used to ____

separate proteins

Phosphorylation usually occurs on what amino acids?

serine (s), threonine (t), tyrosine (y), and histidine (h)

The metatarsals, which are small bones in the hands and feet, exemplify ___ bones

short

____ curves are characteristic of enzymes that display cooperative binding. Through this mechanism, the enzymes affinity for its substrate increases drastically once substrate has been bound to one or more subunits. As a result, the kinetic curve is S-shaped with a sharp increase near the middle

sigmoidal

____ mutations do not alter the the function of the protein produced because of the degeneracy of the genetic code

silent

in _____, the beads in the column are permeated by small pores . While the small proteins can fit through these pores and become trapped in the beads, large molecules cannot; for this reason, larger proteins tend to elute relatively quickly

size exclusion chromatography

Thin and thick filaments do not contract themselves but instead ____relative to one another

slide

Neonates receive maternal antibodies through breast milk. They are ___ to mount a immune response during the first few days of life, but assuming breast feeding continues, they can maintain sufficient immunoglobulin levels to combat an infection

slow

_____ are slow to contract; these fibers have high concentration of myoglobin to supply the oxygen needed for aerobic respiration

slow oxidative fibers

The lower the fluidity of the membrane the ____ its component lipids are able to move

slower

Capillaries experience the ____blood flow of all circulatory vessels. Blood must travel slowly through capillaries to maximize gas and nutrient exchange.

slowest

After reaching an optimum pH, the activity of a particular enzyme will decline. This explains why enzymes from the stomach, such as pepsin, do not work efficiently in the ____, where the pH is higher due to bicarbonate secretion

small intestine

The parasympathetic nervous system promotes "rest and digest" functions. Therefore, it would tend to dilate blood vessels in digestive organs like ___ and ___. Since the goal of the parasympathetic system is to free up blood flow to digestive organs, it would constrict, not dilate blood vessels in the skeletal muscle

small intestine and stomach

The blood brain barrier is permeable only to ______

small, hydrophobic molecules or those with a specific transporter

Peripatric speciation is a subtype of the concept, allopatric speciation and occurs specifically when one of the two populations is ___ than the other

smaller

Arteries, arterioles, veins, and venules contain walls with a layer of _____.

smooth muscle

____ blotting identifies particular DNA sequences

southern blotting

Acidity is determined by the ____ of the CB. The more stable the the conjugate, the stronger the original acid

stability

Cholecystokinin acts in the small intestine upon the entry of food into duodenum from the stomach. This peptide hormone functions to aid a series of processes involved in digestion. Among these are ____,____.____ and ____

stimulating pancreatic acinar cells to release digestive enzymes, stimulating feelings of satiety (fullness) to suppress hunger, inhibiting stomach emptying, and lowering gastric acid secretion

The human epidermis can be described as ____epithelium. The term "stratifed" denotes the multi-layered nature of the tissue

stratified squamous

Complex ll receives electrons from____ and transfers electrons to Coenzyme Q

succinate

Activation of ____ triggers production of a cytokine cocktail that inactivates other immune cells, including cytokine T cells and NK cells that are vital for tumor elimination

suppressor T cells

Many biological processes depend on a very high _____. The large number of tiny alveolar sacs in the lungs, the presence of microvilli in the small intestines, and the folding of the inner mitochondrial membrane are all examples in which an increased surface area allows for biological processes to take place more quickly

surface area to volume ratio

transformation occurs when bacteria pick up genetic material from their ____

surroundings

Mutualism is a

symbiotic relationship in which both parties benefit

Enzymes that are involved in anabolic pathways (ex: A+B-->C) typically include _____ in their names

synthase, synthetase or polymerase

PTH promotes the movement of calcium from bone tissue to ____

systemic circulation

_____ are driven by the tendency of hydrophobic residues to bury themselves inside the protein away from the aqueous environment, as well as side chain interactions such as hydrogen bonding and disulfide bridges

tertiary structures

A ___ is the mating between an unknown individual and a homozygous recessive individual for the traits in question. If any of the offspring demonstrate the recessive phenotype we can conclude the unknown organism was heterozygous for the trait

testcross

Example of x-linked recessive

the mother in the first generation expresses the trait as do 2 of her sons except for 1. However, none of the woman's daughters have the condition, although they must be carriers. This explains how the male in the third generation acquired the trait even though none of his parents display it. The fact that more male than female offspring express this trait, coupled with the idea that males never seem to inherit it from their fathers, is a clear hallmark of a x-linked condition

free energy is a _____ property and not a ___

thermodynamic; kinetic

Gram-negative bacteria do posses a cell wall. This structure is simply ____than the wall present in Gram-positive

thinner

Activators upregulate ____ by binding to either operators (in prokaryotes) or enhancers (in eukaryotes)

transcription

___ is the process where genes are transferred by a virus

transduction

____ is the process by which DNA is transferred from one bacterium to another by a virus

transduction

____ is the genetic alteration of a cell resulting from direct uptake and incorporation of exogenous genetic material from its surrounding through the cellular membrane

transformation

Enzymes catalyze reactions by lowering the activation energy. Enzymes do this by having an active site that bonds favorably with the _____ of the molecule

transition state

Stop codons are involved in the termination of ___, not ___

translation; not transcription

______ proteins enter the endomembrane system by docking at the rough ER. This is facilitated by using a single sequence

transmembrane

A ____ is the result of a mutation that substitutes a purine for a pyrimidine and a pyrimidine for a purine

transversion

_____ are esters derived from glycerol and 3 fatty acids

triacylglycerols

Calcium binds troponin which causes _____ to stop blocking the binding site on the thin filaments for myosin.

tropomyosin

During initiation of of muscle contraction, free Ca 2+ in the cytosol binds binds to _____ which pulls tropomyosin away from actin's myosin-binding site. This allows myosin to bind to actin.

troponin

Proenzymes are inactive protein products that must be cleaved or otherwise modified before they can be activated. Examples include: ____,____and____

trypsinogen, chymotrypsinogen and pepsinogen

enteropeptidase cleaves _____ into __ in the small intestines

trypsinogen; trypsin

_____ normally inhibits growth or arrest the cell cycle

tumor suppressor protein

Divergent evolution

two closely related lineages gradually become more dissimilar

____ bind their target enzymes only when the substrate is first bound to the enzyme

uncompetitive inhibitors

Liver and kidney convert ammonia to urea through combination with CO2. Urea can then be excreted in the ___

urine

integrating bacterial DNA into a viral vector is one method of creating a ____

vaccine

____ an antidiuretic hormone, serves to increase water reabsorption by inducing translocation of aquaporins into the cells of the collecting duct.

vasopressin

Irregular bones are names that do not fit the classification for either short, long or flat bones. Examples include the ____ or the ____

vertebrae or the mandible

______ are typical in dominant traits where the phenotype is very likely to show up in every generation

vertical pattern of inheritance

___ are unique in that they occupy a gray area between living and non-living

viruses

____ is mainly obtained from sunlight and modified in the kidneys and liver

vitamin D

Which of the following vitamins mainly serves as an antioxidant? a) vitamin A b) vitamin B c) vitamin D d) vitamin E

vitamin E is most well known for its anti-oxidizing capabilities vitamin A plays a role in as a regulator of tissue growth The B vitamins are the largest group of vitamins and function in the production of multiple cofactors. Biotin is a B vitamin that acts as an important coenzyme while riboflavin serves as a structural part of the cofactors FMN and FAD Vitamin D is known for its influence on bone growth and calcium regulation

_____ allow calcium cations to flow into the cardiomyocytes, delaying repolarization of these cells; this produces a long plateau at a relatively depolarized voltage

voltage gated Ca 2+ channels

The descending loop of henle is permeable to ___ but not salt. As the filtrate progresses further down the limb, water will continue to flow outward as the filtrate enters an increasingly hypertonic enviornment

water

when action potentials occur at a high frequency within motor neurons, their effects accumulate in a process termed ____

wave summation

The production of steroid hormone is a complex process that involves modification in the smooth endoplasmic reticulum and eventual secretion via the Golgi apparatus. Based on this fact, cells of which of these organs are most likely to posses a enlarged smooth endoplasmic reticulum? a) anterior pituitary b)the adrenal medulla c)the adrenal cortex d)the pancreas

we are basically looking for an organ that is known to secrete steroid hormone. The adrenal cortex synthesizes and releases aldosterone and cortisol, both steroids. The anterior pituitary secretes a variety of hormones but all are peptides The adrenal medulla releases epinephrine and norepinephrine. These two catacholamines are tyrosine derivatives

Buffer solutions which are intended to resist changes in pH consists of a ______

weak acid or a weak base and its corresponding salt

____ is a technique in which proteins are run on a gel using electrophoresis, transferred to a nitrocellulose membrane, and washed with an antibody marker. It is typically used to determine the identity of particular proteins present in a sample and can sort these proteins by molecular weight

western blotting

Why does the polymerization occur more rapidly at the (+) end of an actin polymer than at the (-) end?

while the rate of polymerization or depolymerization depends of the surrounding actin concentration, the (+) end tends to grow much more quickly than the (-) end. (+) actin is bound to ATP while (-) actin is bound to ADP The (+) terminal displays significantly more powerful intermolecular interactions than the (-) actin. When the (+) actin polymerizes, ATP is cleaved to become ADP. The ADP-bound actin at the minus end of the polymer interacts much more weakly, promoting slower, if any polymerization

DNA contains a deoxyribose sugar, while RNA is formed with a ribose sugar, a sugar _____. This small difference makes RNA ____

with a free 2' hydroxyl group; more reactive and unstable and for this reason it rapidly degrades once exiting the nucleus

_____ are neutral molecules that contain at least one positive and one negative charge

zwitterion


Conjuntos de estudio relacionados

3: Sovereignty, States & Constitutional Law

View Set

Business Ethics Now Chapter 1 Terms

View Set